Vous êtes sur la page 1sur 67

Exercices sur les intégrales généralisées

1. Convergence et calcul d’intégrales généralisées.


2. Intégrales généralisées et séries.
3. Intégrales généralisées et sommes de Riemann.
4. Intégration des relations de comparaison.
5. Intégrales semi-convergentes.
6. Espaces fonctionnels.
7. Convergence monotone, convergence dominée
8. Intégration terme à terme des séries.
9. Intégrales dépendant d’un paramètre.
10. Intégrales eulériennes.
11. Transformation de Laplace.
Pierre-Jean Hormière
____________

« Je mets beaucoup d’ordre dans mes idées. Ça ne va pas tout seul.


Il y a des idées qui ne supportent pas l’ordre et qui préfèrent crever.
À la fin, j’ai beaucoup d’ordre et presque plus d’idées. »
Norge
Cher Norge, ce n’est pas bien du tout de vous moquer de ceux qui mettent de l’ordre dans leurs
idées ! Ne seriez-vous pas un peu… belge sur les bords ? Pour ceux qui l’ignorent, la Belgique est
cet improbable petit pays qui est resté plus d’un an sans gouvernement tout en réalisant 2,7 % de
croissance, ceci expliquant peut-être cela… Et de surcroît, en sus d’être belge, ne seriez-vous pas
un peu poète ? Poète belge ! Ah ! vous ne tiendriez pas de tels discours si Napoléon avait gagné la
bataille de Waterloo : la Belgique serait un département français, et vous auriez fait de solides
études mathématiques dans nos glorieuses classes préparatoires. On y apprend à classer ses idées,
afin de diriger le monde, et non à finir antiquaire à Saint-Paul de Vence !

1. Convergence et calcul d’intégrales généralisées.

Exercice 1 : Soient I un intervalle de R, f, g, h trois fonctions réglées de I dans R telle que f ≤ g ≤ h.


Montrer que si f et h sont intégrables, il en est de même de g.

Solution :
Nous avons f ≤ g ≤ h , donc − h ≤ −g ≤ − f . Par conséquent, | g | = max(g, −g ) ≤ max( h, −f ).
Or max( h, − f ) = 1 [ h – f + | h + f | ] est intégrable.
2

+∞
Exercice 2 : Convergence et calcul de I(a, b) = ∫0
dx
(x+a)(x+b)
( a et b > 0 ). Continuité de I ?

Solution :
La fonction f(x) = 1 est continue positive sur R+, et O( 1 ) au V(+∞), ou ≤ 1 , donc
(x+a)(x+b) x² x²
intégrable. Pour calculer I(a, b), décomposons la fraction en éléments simples.
On obtient, si a ≠ b : 1 = 1 ( 1 − 1 ) (*).
(x+a)(x+b) b−a x+a x+b
1 ( +∞ dx − +∞ dx ), mais
b−a ∫0 x+a ∫0 x+b
Attention ! Ne pas écrire I(a, b) =
A A
I(a, b) = limA→+∞ 1 ( ∫ dx − ∫ dx ) = limA→+∞ 1 (ln(A + a) − ln a − ln(A + b) + ln b )
b−a 0 x+a 0 x+b b−a
= limA→+∞ 1 ( ln A+ a a
− ln ) = lnb−ln a .
b−a A+b b b−a
Pour calculer la limite, il y a intérêt à solidifier les logarithmes en un seul bloc. Comme la somme
des résidus est nulle, la limite en l’infini est nulle. Si a = b, on trouve, I(a, a) = 1 .
a

Conclusion : Pour a et b > 0, I(a, b) = lnb−lna si a ≠ b , I(a, a) = 1 .


b−a a
Montrons que la fonction (a, b) → I(a, b) est continue sur R*+×R*+.
Cela vient de ce que les hypothèses (H1), (H2) et (H3) du théorème de continuité des intégrales à
paramètres s’appliquent. En particulier (H3) :
Pour a ≥ α > 0 et b ≥ β > 0, 0 ≤ 1 ≤ 1 , majorante intégrable…
(x+a)(x+b) (x+α)(x+β )

+∞ +∞
Exercice 3 : Convergence et calcul de I = ∫
0
dx
(x+1)(x+2)(x+3)
et J = ∫
0
dx
(x+1)²(x+2)²(x+3)²
.

Solution :
2
Chacune des fonctions intégrées f et g est continue > 0 et O(1/x ) au V(+∞), donc intégrable.
Pour calculer I et J, décomposons f et g en éléments simples.
Décomposons 1 en éléments simples.
(x+1)(x+2)(x+3)
On obtient : 1 = 1 1 − 1 + 1 1 (*).
(x+1)(x+2)(x+3) 2 x+1 x+2 2 x+3
+∞
dx − +∞ dx + 1 +∞ dx , mais
2 ∫ x+1 ∫0 x+2 2 ∫0 x+3
Attention ! Ne pas écrire I = 1
0
A A
dx + 1 A dx
2 ∫0 x+1 ∫0 x+2 2 ∫0 x+3
I = limA→+∞ 1 dx −

= limA→+∞ 1 ln(A + 1) − ln(A + 2) + 1 ln(A + 3) + ln 2 − 1 ln 3


2 2 2
(A+1)(A+3)
= limA→+∞ ln + ln 2 − 1 ln 3 = ln 2 − 1 ln 3
A+2 2 2
Pour calculer la limite, il y a intérêt à solidifier les logarithmes en un seul bloc. Comme la somme
des résidus est nulle, la limite en l’infini est nulle.
1
Pour calculer J, élévons (*) au carré. Il vient :
(x+1)²(x+2)²(x+3)²
= 1 1 + 1 + 1 1 − 1 − 1 + 1 1 .
4 (x+1)² (x+2)² 4 (x+3)² (x+1)(x+2) (x+2)(x+3) 2 (x+1)(x+3)
Il reste à intégrer chaque terme…
+∞ +∞
La situation est analogue au calcul de ∑ 1
n =1 (n+1)(n+2)(n+3)
et ∑ (n+1)²(n+12)²(n+3)² .
n =1

> f:=1/((x+1)*(x+2)*(x+3));convert(f,parfrac,x);

2
int(f,x);int(f,x=0..infinity); 1
1 1 1 1 2
f := − +
(x + 1) ( x + 2) (x + 3) 2 x+1 x+2 x+3
1 1 1
ln( x + 1 ) − ln( x + 2 ) + ln( x + 3 ) − ln( 3 ) + ln( 2 )
2 2 2
> g:=f^2;convert(g,parfrac,x);int(g,x);int(g,x=0..infinity);
1 3
1
g := 1 1 3 1 1 4 4
( x + 1 ) ( x + 2 )2 ( x + 3 )2
2 − + + +
4 ( x + 1 )2 4 x + 1 ( x + 2 )2 ( x + 3 )2 x + 3

1 1 3 1 1 1 3 3 5
− − ln( x + 1 ) − − + ln( x + 3 ) − ln( 3 ) +
4 x+1 4 x+2 4 x+3 4 4 6

+∞ +∞
Exercice 4 : Nature, calcul des intégrales ∫ 0
dx
(x+1).(x+ 2)...(x+ n)
et ∫
0
dx
(x+1)².(x+ 2)²...(x+ n)²
.

Solution : Notons In et Jn ces intégrales respectives, Fn et Gn les fonctions intégrées.


Il est clair que In existe pour n ≥ 2, Jn pour n ≥ 1.
n (−1)k −1 n
Fn(x) = ∑
k =1 x
Ak
+ k
, où Ak =
n!
k Cnk ; on a ∑A
k =1
k = 0 ( car x.Fn(x) → 0 quand x → +∞ ).
n

n! ∑
En solifidifiant les logarithmes comme ci-dessus, il vient : In = 1 (−1) C .k.lnk . k k
n
k =1
n n
Gn(x) = ∑( xB+k
k =1
k
+
Ck
(X +k)²
) , il vient aussitôt Jn = ∑(Ck
k =1
k
− Bk ln k )

2
Or Ck = k² ( Cnk ) par les techniques habituelles, mais il n’est pas facile d’obtenir Bk .
(n!)²
n
∑ (x+k)²
(Ak )²
∑ (X +Ap)(.AX +q) …
2 p q
Mieux vaut noter que Gn(x) = Fn(x) = +2
k =1 p <q

Remarque : On peut obtenir des équivalents des suites (In) et (Jn) à l’aide de la méthode de Laplace.

+∞ +∞
Exercice 5 : Convergence et calcul de I = ∫ dx et J =
−∞ x4 +1 ∫ −∞
x² .dx .
x4 +1

Solution : L’intégrabilité des fonctions continues positives f(x) = 1 et g(x) = x² ne pose


x4 +1 x4 +1
aucun problème : elles sont toutes deux O(1/x²) au V(±∞).
1ère méthode : on peut les calculer séparément par calcul des primitives.
> f:=1/(x^4+1);g:=x^2/(x^4+1);
convert(f,parfrac,x,sqrt(2));convert(g,parfrac,x,sqrt(2));
1
(2 + x 2 )
1 1 −2 + x 2 4
f := − +
x +1
4 4 x2 − x 2 + 1 x2 + x 2 + 1

x2 1 x 2 1 x 2
g := −
x4 + 1 4 x −x 2 +1 4 x +x 2 +1
2 2

> Int(1/(x^4+1),x)=int(1/(x^4+1),x);Int(x^2/(x^4+1),x)=int(x^2/(x^4+1),x);

3
⌠ 1 x +x
2
2 +1 1
 1 1
 4 dx = 2 ln 2 +
 2 arctan( x 2 + 1 ) + 2 arctan( x 2 − 1 )

x + 1 8 x −x 2 +1 4 4

⌠ x2 x −x
2
2 +1 1
 1
 +
1
 4
 x + 1 dx = 8 2 ln x 2 + x 
2 +1 4
2 arctan( x 2 + 1 ) +
4
2 arctan( x 2 − 1 )
 

> Int(1/(x^4+1),x=-infinity..infinity)=int(1/(x^4+1),
x=-infinity..infinity);Int(x^2/(x^4+1),x=-infinity..infinity)
=int(x^2/(x^4+1),x=-infinity..infinity);
∞ ∞
⌠ ⌠
 x2
 1 1
dx = π 2  4
1
 4
  x + 1 dx = 2 π 2
 x + 1 2 
⌡−∞
⌡−∞
2ème méthode : on peut les calculer simultanément.
Tout d’abord, elles sont égales, car le changement de variable y = 1/x donne :
+∞
dx = 2 +∞ dx = 2 +∞ y².dy = +∞ y².dx
I= ∫−∞ x4+1 ∫0 x4+1 ∫0 y4+1 ∫
−∞ y4 +1
= J.

Calculons I + J au moyen du changement de variable u = x – 1/x :


+∞ 1+
1
+∞
+ +∞
+
I + J = ∫ 4 .dx = 2 ∫ 4 .dx = 2 ∫ ² .dx = 2 +∞ du = 2 Arctan u

−∞ x +1
1 x²
0 x +1
1
0
x
x2 + 1
∫0 u2+2 2
+∞
0 =π 2.

Par conséquent I = J = π 2 .
2
t
Remarque : Arnaudiès suggère le changement de variable x = e dans I et J.
3ème méthode : intégration complexe. (hors programme)
Soit γ le lacet obtenu en parcourant le segment [− R, R] et le demi-cercle de centre O et de rayon R
situé dans le demi-plan Re z > 0, parcouru dans le sens trigonométrique (R > 1).
Calculons de deux façons A(R) = ∫γ zdz+1 .
4

+R π iReiθ dθ +∞
D’une part, A(R) = ∫−R xdx
4 +1
+ ∫0 R4e4iθ +1
→ ∫
−∞
dx quand R → +∞
x4 +1
π iReiθ dθ π
Rdθ = πR → 0 quand R → +∞.
car | ∫0 R4e4iθ +1 | ≤ ∫
0 R4 −1 R4 −1
D’autre part, si α = exp(iπ/4), 1 = −1 ( α + iα + −α + −iα ) .
z 4 +1 4 z −α z −iα z +α z +iα
donc : A(R) = −1 2iπ ( α.I(γ, α) + iα.I(γ, iα) − α.I(γ, −α) − iα.I(γ, −i.α) ) .
4
Ces indices valant respectivement 1, 1, 0 et 0, A(R) = −1 2iπ α ( 1 + i ) = π 2 .
4 2
+∞
Exercice 6 : Montrer que ∫
0
ln x .dx converge et vaut 0.
1+ x²

Solution : 1) Convergence.
La fonction f(x) = ln x est continue sur ]0, +∞[, négative sur ]0, 1], positive sur [1, +∞[.
1+ x²
Au V(0+), f(x) ∼ ln x, qui est intégrable.
Au V(+∞), f(x) ∼ ln x = O( 13/ 2 ), donc f est également intégrable.
x² x

4
+∞
2) Calcul. Bien que f ne se primitive pas élémentairement, on peut calculer I = ∫0
ln x .dx .
1+ x²
Le changement de variable y = 1/x donne I = − I, donc I = 0.
1/ε
Plus précisément, le même changement de variable donne ∫ε ln x .dx = 0 pour tout ε > 0.
1+ x²
3) Maple donne ceci, qui intrigue :
> int(ln(x)/(1+x^2),x);
1 1 1 1
− I ln( x ) ln( 1 + I x ) + I ln( x ) ln( 1 − I x ) − I dilog( 1 + I x ) + I dilog( 1 − I x )
2 2 2 2
> int(ln(x)/(1+x^2),x=0..infinity);
0
> int(ln(t)/(1+t^2),t=1..x);
1 1 1 1
− I ln( x ) ln( 1 + I x ) + I ln( x ) ln( 1 − I x ) − I dilog( 1 + I x ) + I dilog( 1 − I x )
2 2 2 2
+ Catalan

+3
Exercice 7 : Nature et calcul de l’intégrale ∫−2
ln x.dx .
+3 −ε +3
Nature de l’intégrale ∫−2
dx ? Trouver lim
x ε→0+ ∫ −1
dx +
x ∫ ε dxx . Explications ?
+

Solution :
La première intégrale est impropre en 0, mais converge, car ln |x| est intégrable au V(0).
+3 3 2
∫−2
ln x.dx = ∫ ln x.dx +
0 ∫ ln x.dx = 3.ln 3 – 3 + 2.ln 2 – 2 = ln(4.27) – 5.
0
La fonction 1/x n’est intégrable ni sur ]0, 3], ni sur [−2, 0], par conséquent la seconde intégrale
diverge.
Cependant, elle converge en un sens affaibli :
−ε +3
∫ dx +
−2 x ∫ ε dxx
+
= ln(ε) − ln 2 + ln 3 − ln(ε) = ln 3 – ln 2.

Variante : par imparité,


−ε
dx + +3 dx = −ε dx + +2 dx + +3 dx = +3 dx = ln 3 – ln 2.
∫−2 x ∫+ε x ∫−2 x ∫+ε x ∫+2 x ∫+2 x
+3 +3
On dit que ∫ dx converge en valeur principale de Cauchy, et l’on note v.p. ∫ dx = ln 3 – ln 2.
−2 x −2 x
+∞
Exercice complémentaire : calculer v.p. ∫ dx .
0 (x−1)(x−2)(x−3)

+1 b
Exercice 8 : Convergence et calcul des intégrales ∫−1
dx et
1− x² ∫a
dt
(b−t)(t −a)
(a < b)

Solution :
La fonction x → 1 est continue positive et non bornée sur ]−1, 1[. L’intégrale est donc
1− x²
impropre en +1 et −1. Mais on dispose d’une primitive élémentaire :
dx = Arcsin d – Arcsin c, donc à la limite +1 dx = Arcsin 1 – Arcsin –1 = π.
d
∫c 1−x² ∫−1 1−x²
Le changement de variable t = a+b + b−a .u ramène aussitôt la seconde intégrale à la première :
2 2

5
b +1

a
dt
(b−t)(t −a)
= ∫ −1
du = π.
1−u²

Exercice 9 : Existence et calcul des intégrales ( n ∈ N ) :


+∞ 1 +∞ +∞
In = ∫ 0
t n.e−t.dt , Jn = ∫ (−lnu)n.du , Kn =
0 ∫ dx , Ln =
−∞ (x²+1)n ∫
−∞
dt .
chnt

Solution :
1) Cet exercice peut être traité de deux façons légèrement différentes.
x
1ère approche. Posons, pour tout x, Fn(x) = t n.e−t.dt . ∫0
−x
F0(x) = 1 − e → 1 quand x → +∞, donc I0 converge et vaut 1.
n −x n −x
Une intégration par parties donne Fn(x) = − x e + n Fn−1(x). Or x e → 0 en +∞.
Si Fn−1(x) a une limite In−1 en +∞, Fn(x) a aussi une limite, et cette limite vaut In = n.In−1.
Ainsi, toutes les intégrales convergent par récurrence, et I0 = 1 et In = n.In−1 impliquent In = n!.
2ème approche. Montrons d’abord que toutes ces intégrales convergent.
n −t 2 n+2 −t
En effet, 0 ≤ t e ≤ 1/t pour t assez grand, car t e → 0 en +∞.
n −t −t/2 n −t/2
Ou bien 0 ≤ t e ≤ e pour t assez grand, car t e → 0 en +∞.
2 −t/2
Or les fonctions tests 1/t et e sont intégrables sur [1, +∞[.
n −t
Dès lors, une ipp donne, pour n > 0 : In = [− t e ] 0+∞ + n.In−1 = n.In−1 .
Comme I0 = 1, on conclut par récurrence que In = n !.
2) On peut étudier des intégrales directement : existence et IPP.
1 +∞
On peut aussi noter que le chgt de variable t = ln u donne Jn = ∫ (−lnu)n.du =
0 ∫ 0
t n.e−t.dt = n!
+∞
3) Kn = ∫ −∞
dx est l’intégrale impropre en ±∞ de
(x²+1)n
1 , fonction continue positive.
(x²+1)n
Par parité, il suffit de se placer en +∞. Or 0 ≤ 1 ∼ 12n qui est intégrable ssi n > 0.
(x²+1)n x
Il est clair que K1 = π. Une intégration par parties donne 2n Kn+1 = (2n – 1)Kn .
(2n−2)!
Donc Kn = 2n−3 2n−5 … 1 K1 = 2n−2 π.
2n−2 2n−4 2 2 (n−1)!²
Remarque : Les Kn sont des intégrales de Wallis, car le changement de variable x = tan θ, ou plutôt
+π / 2
θ = Arctan x, donne Kn = ∫π
− /2
cos2n−2θ.dθ = 2W2n−2.
+∞
4) Ln = ∫ −∞
dt est l’intégrale impropre en ±∞ de 1 , fonction continue positive.
chnt chnt
1 ∼ 2 , fonction intégrable.
n
Par parité, il suffit de se placer en +∞. Or 0 ≤
chnt ent
+∞ +∞
d(sht)
Ln existe pour n ≥ 1. Une intégration par parties Ln = ∫ cht.dt =
−∞ chn+1t ∫
−∞ chn +1t
=…

donne (n + 1).Ln+2 = (n + 2).Ln . Tout revient à calculer L1 et L2.


Remarques : i) La situation est analogue à celle des intégrales de Wallis.
D’ailleurs Ln est une intégrale de Wallis, si l’on passe par la transformation de Gudermann.

6
+π / 2
En effet le changement de variable y = Arctan (sh x) donne Ln = ∫π
− /2
cosn−1 y.dy = 2Wn−1.
ii) La suite (Ln) tend en décroissant vers 0 par convergence dominée.
La méthode de Laplace fournit, quant à elle, un équivalent de Ln :
+∞ +∞
2π .
Ln = ∫
−∞
e−n.ln(cht).dt ∼ ∫
−∞
e−nt² /2.dt =
n

1 ln(1−t²)
Exercice 10 : Convergence et calcul de ∫ 0 t²
.dt .

ln(1−t²)
Solution : 1) C’est l’intégrale impropre en 0 et 1 de la fonction f(t) = continue et < 0.

En 0+, f(t) → − 1 : l’intégrale est faussement impropre.
2
En 1−, f(t) ∼ ln(1 − t ) ∼ ln(1 − t), qui est intégrable.
ln(1−t²) ln(1−t²)
2) Intégrons par parties : ∫ .dt = − + ln | t −1 | .
t² t t +1
1
La limite de cette primitive en 0 étant nulle ( f est d’ailleurs C sur [0, 1[ ) , intégrons sur [0, 1 − ε] :
ln(1−t²) ln(1−(1−ε)²) ln(2−ε)
+ ln | −ε | = − lnε −
1−ε
∫0 t²
.dt = −
1−ε 2−ε 1−ε 1−ε
+ ln ε − ln(2 − ε) → −2 ln 2 ,

car − lnε + ln ε = − ε.lnε → 0 quand ε → 0.


1−ε 1−ε
3) Autre approche, fort différente :
t4 t6
Développons f en série entière sur [0, 1[ : f(t) = − 1 − t² − − −…
4 2 3
ln(1−t²) +∞
∑ n(21n−1)
1
En intégrant terme à terme cette série, il vient formellement : ∫ .dt = − .
0 t² n =1
N N
Or ∑
n =1 n (2
1
n−1)
= ∑(2n2−1− n1) =
n =1
2 ( H2N – 1 HN ) − HN = 2 H2N – 2 HN → 2 ln 2.
2
L’intégration terme à terme est justifiée, soit par associativité de bornes supérieures, soit par appli-
cation du théorème un < +∞.∑∫
1 1
Exercice 11 : Nature et calcul des intégrales : ∫
0
dx
1− x + 1+ x
, ∫0
dx
1− x² + 1+ x²
.

Solution : [ Mines 1982, X 1992, Centrale PC 2010, etc. ]


1
I= ∫ dx
1− x + 1+ x
0
n’est pas une intégrale impropre. Pour la calculer, deux méthodes :
π / 4 sin(2θ).dθ
1ère méthode : poser x = cos(2θ), ou plutôt 2θ = Arccos x . Il vient : I = 2 ∫ .
0 sinθ +cosθ
Les règles de Bioche ne s’appliquent pas, mais on peut exploiter la symétrie sin-cos en notant que :
π /4 sin(2θ).dθ −cos(2ϕ)
π /2 π /22sin ²ϕ −1 π /2 dϕ
I = ∫ 0 sin(θ +π / 4)
= ∫π /4 sinϕ .dϕ = ∫π /4 sinϕ .dϕ = 2 − ∫π /4 sinϕ
.

= 2 − ln( tan
ϕ)
2 + ln tan π = 2 + ln( 2 − 1 ).
π /2 =
π /4
2 8
2ème méthode : Ecrivons I = ∫ 1+ x − 1− x .dx = ∫ 1+ x .dx − 1− x .dx …
1 1 1

0 2x 0 2x ∫ 0 2x
Mais attention, cette idée est fausse, car les deux intégrales divergent.

7
1+ x − 1− x .dx = lim
1 1
1+ x .dx − 1
1− x .dx .
Ecrivons donc I = limα→0+
2x ∫α α→0+ ∫α
2x ∫α 2x
Calculons séparément ces deux intégrales en posant y = 1± x :
1
1+ x .dx = 2 y² .dy = y + 1 ln| y−1 | 2 ,
∫α 2x ∫ 1+α y²−1 2 y+1 1+α
1
1− x .dx = 0 y² .dy = y + 1 ln| y−1 | 0 .
∫α 2x ∫ 1−α y²−1 2 y+1 1−α
1
1+ x − 1− x .dx = 2 + 1 ln 2 −1 − 1−α − 1 ln 1+α −1 + 1−α + 1 ln | 1−α −1 |
∫α 2x 2 2 +1 2 1+α +1 2 1−α +1
→ 2 + 1 ln 2 −1 , car ln( 1+α +1 1− 1−α ) → 0 quand α → 0.
2 2 +1 1+α −1 1−α +1
1
1+ x − 1− x .dx par développement en série entière.
3ème méthode : calculer I = ∫
0 2x
Avec Maple :
> Int(1/(sqrt(1+x)+sqrt(1-x)),x=0..1)=int(1/(sqrt(1+x)+sqrt(1-x)),x=0..1);
evalf(%);
1

 1 1 1
dx = 2 + ln( 2 − 1 ) − ln( 1 + 2 )


 1+x + 1−x 2 2
⌡0
.5328399754 = .5328399746
> Int(sqrt(2)*sin(2*t)/(sin(t)+cos(t)),t=Pi/4..Pi/2)
=int(sqrt(2)*sin(2*t)/(sin(t)+cos(t)),t=Pi/4..Pi/2);
solve(2*t/(1-t^2)=1,t);simplify(convert(simplify(subs(tan(1/8*Pi)=sqrt(2)-
1,2*(-sqrt(2)*tan(1/8*Pi)+arctanh(1/2*(tan(1/8*Pi)-1)*sqrt(2))
*tan(1/8*Pi)^2+sqrt(2)+arctanh(1/2*(tan(1/8*Pi)-1)*sqrt(2)))
/(1+tan(1/8*Pi)^2))),ln));evalf(%);
1/2 π
⌠ 
2
 2 sin( 2 t )
= −   1 
π −  1  1 
π −    1 
π
 d t 2  2 tan  arctanh  
tan  1  2   8 
tan

 sin( t ) + cos( t )  8  2  8     
⌡1/4 π
2
  
− 2 − arctanh  tan π  − 1  2    1 + tan 1 π 
1 1 
 8  
2  8       
−1 − 2 , 2 − 1
1 4 − 4 2 − 4 ln( − 2 + 2 ) + 2 ln( 2 ) + 2 2 ln( − 2 + 2 ) − 2 ln( 2 )
2 2 −2
.5328399745
> Int(sqrt(1+x)/(2*x)-sqrt(1-x)/(2*x),x=0..1)=int(sqrt(1+x)/(2*x)-sqrt(1-x)
/(2*x),x=0..1);
1
⌠ 1 1+x 1 1−x
 −
1 1
dx = 2 + ln( 2 − 1 ) − ln( 1 + 2 )


 2 x 2 x 2 2
⌡0
> Int(sqrt(1+x)/(2*x),x=0..1)-Int(sqrt(1-x)/(2*x),x=0..1)
=int(sqrt(1+x)/(2*x),x=0..1)-int(sqrt(1-x)/(2*x),x=0..1);
1 1
⌠ 1 1+x ⌠ 1 1−x
 dx −  dx = undefined
 

2 x 
2 x
⌡0 ⌡0
> Int(sqrt(1+x)/(2*x),x=epsilon..1)-Int(sqrt(1-x)/(2*x),x=epsilon..1)
=int(sqrt(1+x)/(2*x),x=epsilon..1)-int(sqrt(1-x)/(2*x),x=epsilon..1);

8
1 1
⌠ 1 1+x ⌠ 1 1−x
 dx − 
1 1
dx = 2 + ln( 2 − 1 ) − ln( 1 + 2 ) − 1 + ε
 

 2 x 
 2 x 2 2
⌡ε ⌡ε
1 1 1 1
− ln( 1 + ε − 1 ) + ln( 1 + 1 + ε ) − I π + 1 − ε + ln( 1 − ε − 1 )
2 2 2 2
1
− ln( 1 + 1 − ε )
2
> f:=epsilon->sqrt(2)+1/2*ln(sqrt(2)-1)-1/2*ln(1+sqrt(2))-sqrt(1+epsilon)-
1/2*ln(sqrt(1+epsilon)-1)+1/2*ln(1+sqrt(1+epsilon))+sqrt(1-epsilon)
+1/2*ln(1-sqrt(1-epsilon))-1/2*ln(1+sqrt(1-epsilon));
series(f(epsilon),epsilon=0,2);
1 1 1
f := ε → 2 + ln( 2 − 1 ) − ln( 1 + 2 ) − ε + 1 − ln( ε + 1 − 1 )
2 2 2
1 1 1
+ ln( 1 + ε + 1 ) + 1 − ε + ln( 1 − 1 − ε ) − ln( 1 + 1 − ε )
2 2 2
 2 + 1 ln( 2 − 1 ) − 1 ln( 1 + 2 )  − 1 ε + O( ε 2 )
 
 2 2  2
1
J= ∫
0
dx
1− x² + 1+ x²
, également non impropre, se traite de la même façon.

∫ dx = ∫ 1+ x² − 1− x² .dx = ∫ 1+ x² .dx − ∫ 1− x² .dx


1− x² + 1+ x² 2x² 2x² 2x²
Ces intégrales se calculent par intégrations par parties ou par changement de variable
1+ x² .dx = − 1 1+ x² + 1 Argsh x , 1− x² .dx = − 1 1− x² − 1 Arcsin x .
∫ 2x² 2x 2 ∫ 2x² 2x 2
1 1
1+ x² .dx − 1
1− x² .dx .
Ecrivons I = limα→0+ ∫α 1−x²dx
+ 1+ x²
= limα→0+ ∫α 2x² ∫α 2x²

Au final, on trouve J = π − 2 + 1 ln(1 + 2 ).


4 2 2
> int(1/(sqrt(1+x^2)+sqrt(1-x^2)),x=0..1);
1 1 1
− 2 − ln( 2 − 1 ) + π
2 2 4
> f:=int(sqrt(1+x^2)/(2*x^2),x)-int(sqrt(1-x^2)/(2*x^2),x);
( 3/2 ) ( 3/2 )
1 ( 1 − x2 ) 1 1 1 ( 1 + x2 ) 1
f := + x 1 − x 2 + arcsin( x ) − + x 1 + x2
2 x 2 2 2 x 2
1
+ arcsinh( x )
2
> subs(x=1,f)-limit(f,x=0);
1 1 1
arcsin( 1 ) − 2 + arcsinh( 1 )
2 2 2

π /2 π /2
Exercice 12 : Convergence et calcul de I = ∫ 0
tan x.dx et J = ∫
0
cos x.ln(tan x).dx .

Solution : • I est l’intégrale impropre en π de la fonction f(x) = tan x qui est continue positive et
2
équivalente à 1 = 1 ∼ 1 au V( π ).
cos x π
sin( − x) π −x
2
2 2
Les changements de variable t = tan x, puis u = t donnent :

9
+∞
t .dt = +∞
2u² .du = π 2 si l’on en croit l’exercice 3.
I= ∫0 t²+1 ∫0 u 4 +1 2
• J est l’intégrale imprope en 0 et π de la fonction f(x) = cos x.ln(tan x) qui est continue, positive
2
sur [ π , π [, négative sur ]0, π ].
4 2 4
3 3 2
Au V(0+), f(x) ∼ ln(tan x) ∼ ln x, car ln(tan x) = ln(x + O(x )) = ln x + ln(1 + O(x )) = ln x + O(x ).
Or ln x est intégrable sur ]0, 1].
Au V( π −), x = π − h, donc f(x) = − sin h.ln(tan h) ∼ − h.ln h ; il y a fausse impropreté.
2 2
π /2
Pour calculer J = ∫ 0
cos x.ln(tan x).dx , intégrons par parties.
Mais pour éviter tout problème, cherchons une primitive de f par parties.

∫ cosx.ln(tan x).dx = sin x.ln(tan x) − ln | tan( 2x + π4 ) | = F(x).


lim x→0+ F(x) = 0 car sin x.ln(tan x) → 0 en 0+ (voir ci-dessus).
En π −, x = π − h, et F(x) = − cos h.ln(tan h) + ln(tan h )
2 2 2
= cos h.ln(cos h) − cos h.ln(sin h) + ln (sin h ) − ln (cos h ) = − cos h.ln(sin h) + ln (sin h ) + o(1).
2 2 2
Or − cos h.ln(sin h) + ln (sin h ) = − cos h.( ln h + O(h)) + ln h + O(h) → − ln 2.
2 2

tan x.dx = π 2 ,
π /2 π /2
Conclusion : ∫
0 2 ∫0
cos x.ln(tan x).dx = − ln 2.
+∞
Variante : le changement de variable t = tan x donne I = ∫
0
lnt .dt .
(1+t²)3/ 2

Exercice 13 : Nature et calcul des intégrales suivantes :


+∞ +∞ +∞ +∞ x3.ln x +∞
x .dx ,
∫0 (x² +1)² ∫
0
dx
(x+1) x² + x+1
, ∫0
dx
1+ x x² +1
, ∫0 (x4 +1)3
.dx , ∫
0
dx
(x+1).x 2/ 3
,

x
1
1− x .dx , 1
∫0 ∫ (x+2) dxx(1− x) .
x(1+ x )
0
1− x

Solution :
+∞ +∞
x .dx =
1) Le chgt de variable t = x donne ∫
0 (x² +1)² ∫ 0
2t².dt .
(t 4 +1)²
Nous voilà ramenés à une fraction rationnelle... Décomposer en éléments simples, passer par la
variable complexe, ou faire appel à Maple…
2) f(x) = 1 est continue positive sur R+, équivalente à 12 au V(+∞), donc intégrable.
(x+1) x²+ x+1 x
2 2
On a x + x + 1 = ( x + 1 ) + 3 : poser x = − 1 + 3 sh t, etc.
2 2 4 2
3) f(x) = 1 est continue positive sur R+, équivalente à 12 au V(+∞), donc intégrable.
1+ x x²+1 x
+∞ +∞
Le changement de variable x = sh t donne ∫ dx = ∫ cht.dt .
0 1+ x x² +1 0 1+ sht.cht

Fraction rationnelle en ch-sh…

10
x3.ln x
4) f(x) = est continue, tend vers 0 en 0+, et est O( 18 ) au V(+∞). Elle est donc intégrable.
(x4 +1)3 x
+∞ x3.ln x +∞
∫ ∫
4 lnu .du .
Le changement de variable u = x donne .dx = 1
0 (x4 +1)3 16 0 (u +1)3
−lnu + 1 du = −lnu + 1 ln u + 1 1 .
Intégrons par parties : ∫ (uln+1u) .du = 3 2(u +1)² 2 ∫ u(u+1)² 2(u +1)² 2 u+1 2 u +1
+∞ x3.ln x
Limite en +∞ : 0, limite en 0+ : 1 . Au final :
2 ∫
0 (x4 +1)3
.dx = − 1 .
32

f(x) = 1 est continue positive sur ]0, +∞[, intégrable car f(x) ∼ 12/3 en 0+, ∼ 14/3 en +∞.
(x+1)x2/3 x x
+∞ +∞
∫ ∫
1/3 dx 3 .du .
5) Le changement de variable u = x donne : =
0 (x+1).x 2/ 3 0 u3+1
3 = 1 + −u+2 = 1 − 1 2u−1 + 3 1 .
u3+1 u +1 u²−u+1 u +1 2 u²−u+1 2 u²−u +1
u +1
Donc : ∫ u 3+1.du = ln + 3 Arctan 2u −1 . Au final :
3
u²−u+1 3
+∞ +∞
3 .du = 2π 3 .

0
dx
(x+1).x 2/ 3
= ∫
0 u3+1 3
x
6) f(x) = 1− x est continue positive sur ]0, 1[, f(x) ∼ 1 en 0+, à 1 en 1 (fausse impropreté).
x(1+ x ) x
1− x
2
Les changements de variable x = sin θ , θ = Arcsin x , puis t = tan θ , donnent :
x
1
1− x .dx = 2 π /2
dθ = 2 +∞
= ... = ln 1+t + Arctan t = π.
∫0 x(1+ x ) ∫0 1+tanθ ∫
0
dt
(1+t²)(1+t) 1+t²
+∞
0
2
1− x
Variante : le changement de variable t = x donne le même résultat.
1− x
7) f(x) = 1 est continue positive sur ]0, 1[, équivalente à 1 en 0+ et à 1 en
(x+2) x(1− x) 2 x 3 1− x
1−, donc intégrable sur ]0, 1[. Le changement de variable 2x – 1 = sin θ (plutôt θ = Arcsin… ) donne
1 1 +π / 2
2.dθ , puis t = tan(θ/2)…
I= ∫0 (x+2) dxx(1− x) = ∫0 (x+2) 21−dx(2x−1)² = ∫π
− /2 5+sinθ
> f:=x->1/((x+2)*sqrt(x*(1-x)));int(f(x),x);int(f(x),x=0..1);
1 1 ( −2 + 5 x ) 6 
6 arctan 
6  12
 −( x + 2 ) + 5 x + 4 
2

1
π 6
6

Exercice 14 : Convergence et calcul de


a +∞ +∞

0
x.dx
(x²+1)(a²− x²)
(a>0) , ∫ dx
− ∞ (x² + a²) x² +b²
( a, b > 0 ) , ∫0
dx
cosa +chx
.

Solution (partielle) :

11
a² dy

2
1) Le changement de variable y = x ramène la 1ère intégrale à I = .
0 2 (y+1)(a²− y)
2
L’intégrale est impropre en a , mais convergente (règle de l’équivalent).
+∞
3) Si a ≡ π (mod 2π), ∫
0
dx est impropre en 0 et en +∞, et divergente sur ]0, 1] (règle de
chx−1
l’équivalent). Sinon, l’intégrale est impropre en +∞, et convergente (règle de l’équivalent).
+∞ +∞ 2ex.dx +∞ +∞
∫0
dx
cosa +chx
= ∫
0 e2x +2ex cosa+1
= ∫1
2.dt
t +2t cosa+1
2
= ∫
1
2.dt
(t +cosa)²+sin²a
( où t = exp x )

Puis l’on pose t + cos a = u.sin a . On trouve au final a , si 0 < a < 2π .


sina

1
Exercice 15 : Existence et calcul des intégrales : I(a) = xa.ln x.dx . ∫ 0

a
Solution : f(x) = x .ln x est continue négative sur ]0, 1] .
Si 0 < a , f(x) → 0 en 0+ ; il y a fausse impropreté.
a +1
Si −1 < a , f(x) = O( 1
1−a ) au V(0+), car x
2 ln x → 0. Or 1 est intégrable sur ]0, 1].
1−a
x 2 x2

∫ε lnxx.dx = − ln2²ε
1
Si a = − 1, f(x) = ln x est non intégrable, car → −∞. A fortiori si a < − 1.
x
Conclusion : I(a) est définie pour a > −1. Une intégration par parties donne alors I(a) = − 1 .
(a+1)²
−t +∞
NB : le chgt de variable x = e donne I(a) = ∫ 0
te−(a+1)t.dt , fournissant un autre angle d’attaque.

π /2 π /2
Exercice 16 : Existence et calcul de I = ∫
0
ln(sint).dt ( On pourra noter que I = ∫0
ln(cost).dt ).

Solution :
La fonction f : t → ln(sin t) est continue négative sur ]0, π ].
2
3 2
Au V(0+), f(t) = ln( t + O(t ) ) = ln t + O(t ) ∼ ln t, donc f est intégrable.
Pour calculer I, notons que :
π /2 π /2 π /2 π /2 sin(2t)
I=
0 ∫ ln(sint).dt =
0 ∫
ln(cost).dt = 1 ∫ ln(sint.cost).dt =
2 0
1
2 ∫
0
ln
2
.dt =
π /2
π π
ln(sinu).du − π ln 2 =
π /2
ln(sinu).du − π ln 2
= 1
2 0 ∫ ln(sin2t).dt − ln 2 =
4
1

4 0 4
1
2 ∫0 4
= 1 I − π ln 2 , par pliage, donc I = − π ln 2 .
2 4 2
Remarque : d’autres méthodes existent, moins astucieuses.

+∞
Exercice 17 : Nature de l’intégrale de Gauss ∫
0
e− x².dx .

Solution : La fonction x → e−x² est continue positive, mais ne se primitive pas élémentairement.
A
On ne peut donc montrer la convergence de l’intégrale en calculant ∫e
0
− x².dx . Nous allons procéder
par comparaison à des fonctions tests connues.

12
1ère méthode : je dis que, pour x assez grand, x ≥ a > 0, 0 ≤ e−x² ≤ 1 , car x2e−x² → 0 quand x → +∞.

Comme 1 est intégrable sur [a, +∞[, e−x² itou.

2ème méthode : je dis que, pour tout x, 0 ≤ e−x² ≤ 1 , car 1 + x2 ≤ e−x² (au fait, pourquoi ?).
x²+1
Comme 1 est intégrable sur [0, +∞[, e−x² itou.
x²+1
3ème méthode : je dis que, pour x ≥ 1, 0 ≤ e−x² ≤ e−x . Comme e−x est intégrable sur [1, +∞[, e−x² itou.
Remarque : On peut néanmoins calculer par divers moyens indirects l’intégrale de Gauss
+∞
π . La méthode la plus classique consiste à considérer
∫0
e− x².dx , et démontrer qu’elle vaut
2
l’intégrale double ∫∫
[0,+∞[²
e−x²− y².dxdy et à la calculer de deux façons. Mais on peut aussi passer par la
variable complexe.

1
Exercice 18 : Nature de l’intégrale ∫ sin(1x).dx .
0

Solution : La fonction x → sin(1/x) est définie sur R*, et n’a pas de limite en 0, car elle oscille au
V(0). L’intégrale proposée est impropre en 0.
1
Faute de pouvoir calculer ∫εsin(1x).dx , on va montrer l’absolue convergence par comparaison.
Le plus simple est de noter que | sin 1 | ≤ 1. Comme la fonction x → 1 est intégrable sur ]0, 1], la
x
fonction x → sin(1/x) l’est également.
+∞ sin y
On peut aussi noter que le changement y = 1/x transforme l’intégrale en ∫1 y²
.dy , qui est
clairement absolument convergente.
Remarques : 1) On peut montrer qu’une fois prolongée arbitrairement en 0, la fonction x → sin(1/x)
n’est pas réglée, mais est Riemann-intégrable sur [0, 1].
1
2) Maple affirme que ∫ sin(1x).dx
0
= sin 1 – Ci(1) ≈ 0.504. Encore faut-il connaître la fonction
« Cosinus intégral » Ci…
> A:=int(sin(1/x),x=0..1);evalf(A);
A := sin( 1 ) − Ci( 1 )
.5040670619
+∞
3) L’intégrale ∫
0
sin 1.dx diverge car 0 < sin(1/x) ∼ 1/x au V(+∞).
x

+∞
Exercice 19 : Nature de l’intégrale ∫0
(1−cos 1 ).dx .
x

Solution : La fonction x → 1 – cos(1/x) est continue positive sur ]0, +∞[.


2
Elle est bornée sur ]0, 1], et O(1/x ) au V(+∞), donc intégrable sur [1, ∞[.
En résumé, elle est intégrable sur ]0, +∞[.
+∞ 1−cos y
Remarques : 1) Le changement de variable y = 1/x transforme l’intégale en ∫
0 y²
.dy , qui est
1−cos y
faussement impropre en 0 et convergente sur [1, +∞[, car 0 ≤ ≤ 2 .
y² y²
2) Maple affirme que :

13
> B:=int(1-cos(1/x),x=0..infinity);evalf(B);
1
B := π
2
1.570796327

+∞ x.sin(1/ x²)
Exercice 20 : Nature de l’intégrale A = ∫0 ln(1+ x)
.dx .

Solution : [ Oral Mines 1980 ]


x.sin(1/ x²)
C’est l’intégrale impropre en 0+ et en +∞ de la fonction f(x) = .
ln(1+ x)
Etudions séparément son intégrabilité au V(+∞) et au V(0+).
• Au V(+∞), f(x) ∼ 1 ≤ 13/ 2 ; par conséquent, f est intégrable.
x3/2.ln x x

• Au V(0+), | f(x) | ≤ x ∼ 1 ; par conséquent, f est intégrable.


ln(1+ x) x
En fait, au V(0+), la fonction f(x) oscille entre x et − x .
ln(1+ x) ln(1+ x)
> with(plots):f:=x->sqrt(x)*sin(1/x^2)/ln(1+x);g:=x->sqrt(x)/ln(1+x);
> p:=plot(f(x),x=0..3,-4..4,thickness=2,numpoints=1000):
q:=plot([g(x),-g(x)],x=0..3,-5..5,color=blue):display({p,q});

+∞ 1/ 2
Exercice 21 : Discuter la nature des intégrales de Bertrand : ∫ 2
dx
x .(ln x)b
a
et ∫
0
dx .
x .ln x
a
b

Solution : La première de ces intégrales est au programme sans être au programme, tout en l’étant…
• Notons F(a, b) la première intégrale. Elle converge ssi a > 1, ou (a = 1 et b > 1), autrement dit ssi
(a, b) > (1, 1) pour l’ordre lexicographique.
C’est l’intégrale impropre en +∞ de la fonction continue et positive x → 1 .
xa.(ln x)b
Le plus simple est de commencer par le cas a = 1. Le changement de variable u = ln x donne :
A ln(A)
∫2
dx
x.(ln x)b
= ∫
ln(2)
du . On sait que cette intégrale converge ssi b > 1.
ub
1 1 a−1 b−2
Si a > 1, je dis que 0 < < pour x assez grand, car x (ln x) → +∞ en +∞.
x .(ln x)
a b x.(ln x)2

14
+∞ +∞
Comme ∫
2
dx converge,
x.(lnx)2 ∫ 2
dx
xa.(ln x)b
converge.

1 a−1 b
Si a < 1, je dis que 0 < 1 < pour x assez grand, car x (ln x) → 0 en +∞.
x xa.(ln x)b
+∞ +∞
Comme ∫
2
dx diverge,
x ∫
2
dx
xa.(ln x)b
diverge.

• Soit G(a, b) la seconde intégrale. Elle converge ssi a < 1 ou a = 1 et b > 1.


1/ 2 +∞
Le changement de variable u = 1/x donne en effet : ∫ 0
dx =
xa.ln x
b ∫2
du .
u 2−a.lnbu
Nous voilà ramenés aux intégrales précédentes. Mais on pouvait aussi rester au V(0+).
+∞
Remarque : l’intégrale ∫0
dx est toujours divergente. Elle est impropre en 0+, 1 et +∞.
x .ln x
a
b

En effet, en vertu de ce qui précède, il y a convergence sur ]0, ½] et [2, +∞[ ssi a = 1 et b > 1.
b ∼
Mais au V(1), 1 1 . Il y a convergence sur [½, 2] ssi b < 1.
x−1
b
x .ln x
a

+∞
Exercice 22 : Discuter la nature de l’intégrale : ∫ 3
dt
t a.(lnt)b.(lnlnt)c
.

Solution :
• Soit F(a, b, c) la première intégrale. Elle converge ssi a > 1, ou a = 1 et b > 1, ou a = b = 1 et c > 1,
autrement dit ssi (a, b, c) > (1, 1, 1) pour l’ordre lexicographique.
Le plus simple est d’étudier le cas a = b = 1. Le chgt de variable u = ln ln t donne
+∞ +∞

3
dt
t.lnt.(lnlnt)c
= ∫ ln(ln 3)
du . L’intégrale converge ssi c > 1.
uc
Si a > 1, ou a = 1 et b > 1, alors fa,b,c(t) ≤ f1,1,2(t) au V(+∞).
Si a < 1, ou a = 1 et b < 1, alors fa,b,c(t) ≥ f1,1,1(t) au V(+∞).
• Soit G(a, b) la seconde intégrale. Elle converge ssi a < 1 ou a = 1 et b > 1.
1/ 2 +∞
Le changement de variable u = 1/t donne en effet : ∫
0
dt =
t a.lnt
b ∫ 2
du .
u 2−a.lnbu
Nous voilà ramenés à des Bertrand classiques. Mais on peut aussi rester au V(0+).

+∞ +∞ +∞ ln(1+t a)
Exercice 23 : Discuter la nature des intégrales : ∫1
dx
xa.(1+ xb)
, ∫ 0
dx
xa.(1+ xb)
et ∫
0 tb
.dt .

Solution :
1) La fonction f(x) = 1 est continue et positive sur [1, +∞[ .
xa(1+ xb)
En +∞, f(x) ∼ 1 si b > 0 ; il y a intégrabilité ssi a + b > 1.
x a +b
f(x) ∼ 1 a si b = 0 et f(x) ∼ 1a si b < 0 ; il y a intégrabilité ssi a > 1.
2x x
+∞
I(a, b) = ∫ 1
dx
xa.(1+ xb)
est définie sur D = { (a, b) ; b > 0 et a + b > 1 } ∪ { (a, b) ; b ≤ 0 et a > 1 } .

15
2) En 0+, f(x) ∼ 1a si b > 0 et f(x) ∼ 1 a si b = 0 ; il y a intégrabilité ssi a < 1.
x 2x
f(x) ∼ 1 si b < 0 ; il y a intégrabilité ssi a + b < 1.
x a +b
+∞
J(a, b) = ∫
0
dx
xa.(1+ xb)
est définie sur D ∩ D’, avec :

D’ = { (a, b) ; b ≥ 0 et a < 1 } ∪ { (a, b) ; b < 0 et a + b < 1 } .

Exercice 24 : 1) Soit f ∈ C(R+, R+) une fonction intégrable. Construire une suite croissante (xn)
+∞
telle que (∀n) ∫xn
f(t).dt ≤ 1n . En déduire qu’existe une fonction g ∈ C(R+, R+) croissante,
2
tendant vers +∞ en +∞, telle que f.g soit intégrable sur R+.
2) Soit f ∈ C(R+, R+) une fonction non intégrable. Montrer qu’il existe une fonction g∈C(R+, R+)
décroissante, tendant vers 0 en +∞, telle que f.g ne soit pas intégrable sur R+.

Solution :

2. Intégrales généralisées et séries.

+∞
Exercice 1 : Soit ∑a
n =0
n une série à termes ≥ 0.

Montrer que la fonction f : R+ → R+ définie par f(x) = an pour n ≤ x < n+1, est intégrable sur R+ si
+∞ +∞ +∞
et seulement si la série ∑an converge, et qu’alors :
n =0

0
f(x).dx = ∑a
n =0
n .

Solution :

Exercice 2 : 1) Indiquer une fonction continue positive et intégrable sur R+, bornée mais ne tendant
pas vers 0 en +∞.
2) Indiquer une fonction continue positive intégrable et non bornée sur R+.
3) Montrer toutefois que si f est continue, positive et intégrable sur R+ et intégrable, 0 est valeur
d’adhérence de f en +∞, autrement dit, il existe une suite (xn) tendant vers +∞ telle que f(xn) → 0.

Solution :
Considérons la fonction continue affine par morceaux f : R+ → R+ définie par :
f(n) = n , f(n ± 1 ) = 0 (n ≥ 1) , f étant affine entre ces points.
n .2 n
n +1/ 2
Alors ∫ 0
f(t).dt = 1 + 1 + ... + 1n = 1 − 1n ↑ 1.
2 2² 2 2
Comme tout segment [a, b] est inclus dans un [0, n + 1 ], f est intégrable sur R+, d’intégrale 1.
2

+∞
Exercice 3 : a) Nature de l’intégrale ∫
0
dx
1+ x2.sin ²x
.

16
+∞ xa.dx
b) Soient a et b > 0. Montrer que l’intégrale ∫ 0 1+ xb.sin ²x
converge ssi b > 2a + 2.

Solution :
1) Analyse. La fonction f(x) = 1 est continue positive sur R+.
1+ x².sin²x
L’encadrement 1 ≤ f(x) ≤ 1 ne conclut pas.
1+ x²
Le graphe de f oscille de l’une à l’autre fonction, donc f n’a pas d’équivalent simple en +∞.
> with(plots):f:=x->1/(1+x^2*sin(x)^2):
p:=plot(f(x),x=0..15,thickness=2,numpoints=2500):
q:=plot(1,x=0..15,color=black):r:=plot(1/(1+x^2),x=0..15,color=blue):
display({p,q,r});

2) Transformons l’intégrale en série : on sait en effet que f intégrable si et seulement si la série


+∞ (n +1)π π
∑u
n =0
n converge, où un = ∫π n
f(x).dx = ∫ 1+(u+nduπ)²sin²u
0
(x = nπ + u).
π π
Encadrons un : ∫ 1+(n+1du
0 )²π².sin²u
≤ un ≤ ∫ 1+n²πdu².sin²u .
0
π π /2
an = ∫ 1+n²πdu².sin²u = 2 ∫ 1+n²πdu².sin²u
0 0
se calcule aisément, la règle de Bioche autorisant t = tan u.

an = 2 ∫
+∞
dt = 2∫
+∞
dt = π ∼ 1.
(1+t²)[1+ n²π²t² ]
0 0 1+(1+n²π²).t² 1+n²π² n
1+t²
+∞ +∞
Par encadrement, un ∼ 1 , donc
n ∑u
n =0
n et ∫0
dx
1+ x2.sin ²x
divergent.

Variante : évitons le calcul exact de an en utilisant l’encadrement ∀u∈[0, π ] 2u ≤ sin u ≤ u.


2 π
π /2 π /2 π /2
2 du

1+(n+1)²π²u²
0
≤ an = 2 ∫ du
0 1+n²π².sin ²u
≤ 2∫ du .
0 1+4n²u²

Un calcul montre que (an) est semblable à 1 . Résultat plus faible, mais qui suffit à conclure.
n
3) Généralisation facile laissée au lecteur.
Références : Valiron p. 114, Ramis p. 102, Couty-Ezra, p. 401, Ovaert 2 p. 630, POX 1984, etc.

+∞
Exercice 4 : Soient a, b > 0. Montrer que l’intégrale ∫0
xa.exp(−xb.sin ²x).dx converge ssi b > 2a + 2.

Solution :
a b 2
1) Analyse. La fonction f(x) = x .exp(−x sin x) est continue et positive sur R+.
a b a
On a l’encadrement x .exp(−x ) ≤ f(x) ≤ x , le graphe de f oscillant de l’une à l’autre.
> with(plots):f:=x->x*exp(-x^3*sin(x)^2);

17
p:=plot(f(x),x=0..15,numpoints=2500,thickness=2):
q:=plot(x,x=0..15,color=black):
r:=plot(x*exp(-x^3),x=0..15,color=blue):display({p,q,r});

2) Transformons l’intégrale en série : on sait en effet que f intégrable si et seulement si la série


+∞ (n +1)π π
∑u
n =0
n converge, où un = ∫π n
f(x).dx = ∫ (nπ +u) .exp(−(nπ +u) .sin²u).du
0
a b ( x = nπ + u ).
π π
Encadrons un : ∫ n π .exp(−((n+1)π) .sin²u).du ≤ un ≤ ∫ (n+1) π .exp(−(nπ) .sin²u).du .
0
a a b
0
a a b

an = ∫ exp(−(nπ) ..sinu).du = 2 ∫ exp(−(nπ) ..sin ²u).du . Or ∀u ∈ [0, π ] 2u ≤ sin u ≤ u.


π π /2
b b
0 0 2 π
π /2 π /2
Donc 2 ∫ exp(−(nπ) .u²).du ≤ an ≤ 2 ∫ exp(−(nπ) .4u² ).du .
b b
0 π² 0
π /2 (nπ)b / 2π / 2
π
∫ exp(−(nπ)b.u²).du = ∫
b/2
Le chgt de variable s = u(nπ) donne 1 e−s².ds ∼ .
0 (nπ)b/2 0 2(nπ)b/ 2
On traite de même la majorante. On en déduit que (an) est semblable à 1b/ 2 .
n
a a 1 .
Comme (nπ) an+1 ≤ un ≤ ((n+1)π) an , (un) est semblable à
nb/ 2−a
+∞
Finalement, ∑u
n =0
n converge ssi b > 2a + 2.

Remarques : 1) La méthode de Laplace fournit même un équivalent de un, car elle affirme que :
π /2 π /2
π

0
exp(−(nπ)b..sin²u).du ∼ ∫0
exp(−(nπ)b.u²).du ∼
2(nπ)b/ 2
.
a b 2 a b
2) Comme x .exp(−x sin x) ≥ x .exp(−x | sin x |), la condition b > 2a + 2 assure la convergence de
+∞
∫0
xa.exp(−xb.sin x).dx . L’exercice suivant creuse cette question.

+∞
Exercice 5 : i) Nature de l’intégrale ∫ 0
exp(−x.sin x).dx .
+∞
ii) Soient a, b > 0. Montrer que l’intégrale ∫
0
xa.exp(−xb.sin x).dx converge ssi b > a + 1.

Solution :
1) Analyse.
C’est l’intégrale impropre en +∞ de la fonction continue et positive f(x) = exp( − x.| sin x | ).
f est une fonction oscillante, car (∀x) exp(−x) ≤ f(x) ≤ 1, le graphe de f oscillant constamment entre
ces deux fonctions.

18
> with(plots):
> p:=plot(1,0..15,color=black):q:=plot(exp(-x),x=0..15,color=blue):
r:=plot(exp(-x*abs(sin(x))),x=0..15,numpoints=2000,thickness=2):
display({p,q,r});

2) Transformons l’intégrale en série : on sait en effet que f est intégrable si et seulement si la série
+∞ (n +1)π π
∑u
n =0
n converge, où un = ∫π
n
f(x).dx = ∫ exp(−(nπ +u).sinu).du
0
(x = nπ + u).
π π
Encadrons un : 0 ≤ ∫ exp(−(n+1)π..sinu).du ≤ un ≤ ∫ exp(−nπ..sinu).du .
0 0

an = ∫ exp(−nπ..sinu).du = 2 ∫ exp(−nπ..sinu).du . Or ∀u ∈ [0, π ] 2u


π π /2
≤ sin u ≤ u.
0 2 π
0
π /2 π /2
Donc 2 ∫ exp(−nπu).du ≤ an ≤ 2 ∫ exp(−2nu).du .
0 0

Ces integrales se calculent et montrent que (an) est semblable à 1 .


n
Comme an+1 ≤ un ≤ an , (un) est également semblable à 1 .
n
Finalement, la série et l’intégrale divergent.
π /2
Remarque : la méthode de Laplace fournit même un équivalent de an = 2 ∫0
exp(−nπ..sinu).du .
π /2
Par concentration de masse en 0, on montre que an ∼ 2 ∫
0
exp(−nπu).du ∼ 2 .

a b
3) Généralisation. La fonction f(x) = x .exp(−x | sin x |) est intégrable si et seulement si la série
+∞ (n +1)π π
∑u
n =0
n converge, où un = ∫π
n
f(x).dx = ∫ (nπ +u) .exp(−(nπ +u) .sinu).du
0
a b ( x = nπ + u ).
π π
Encadrons un : ∫ n π .exp(−((n+1)π) .sinu).du ≤ un ≤ ∫ (n+1) π .exp(−(nπ) .sinu).du .
0
a a b
0
a a b

an = ∫ exp(−(nπ) ..sinu).du = 2 ∫ exp(−(nπ) ..sinu).du . Or ∀u ∈ [0, π ] 2u ≤ sin u ≤ u.


π π /2
b b
0 2 π
0
π /2 π /2
Donc 2 ∫ exp(−(nπ) .u).du ≤ an ≤ 2 ∫ exp(−(nπ) .2u ).du .
b b
0 π 0

Ces integrales se calculent et montrent que (an) est semblable à 1b .


n
a a 1 .
Comme (nπ) an+1 ≤ un ≤ ((n + 1)π) an , (un) est semblable à
nb−a
+∞
Finalement, ∑u
n =0
n converge ssi b > a + 1.

Exercice 6 : Natures des intégrales :


+∞ e2x.dx +∞ +∞ +∞
∫ ∫ ∫π sin ²x .dx ,

x
, sin x .dx , t.e−at E(et).dt (E est la partie entière) .
0 e− x +e3x.sin x 0 x.ln x 0

19
Solution :

+∞ e−t.dt +∞ +∞
Exercice 7 : Natures des intégrales : ∫0
cost
, ∫
0
dx
(1+ x²).sin 2/ 3 x
, ∫π a
dx , (a, b) ∈ R2.
x .sin x
b

Solution : Etudions en détail la première intégrale, tirée de Arnaudiès-Fraysse, t.2, p. 424.


e−t
La fonction f(t) = est continue > 0 sur [0, π [ ∪ ] π , 3π [ ∪ …
cos(t) 2 2 2

Cette fonction possède une infinité d’asymptotes sur R+ ( Ah ! posséder une asymptote !…)
Donc l’intégrale est impropre en tous les nπ + π et bien sûr en +∞.
2
nπ + π
∫π π
2
Chacune des intégrales In = f(t).dt converge en vertu de la règle de l’équivalent.
n −
2

−nπ +π
∫π
2
De plus, le changement de variable t = nπ + u donne In = e f(t).dt .

2

Enfin, la série de terme général In converge en tant que série géométrique.


+π +π +π +π
Par conséquent, I = ∫
+0
2
f(t).dt + ∑e π ∫ π
n≥1
−n

2

2
f(t).dt = ∫ +0
2
f(t).dt + 1
eπ −1 ∫π −
2

2
f(t).dt .
x ∞
Remarque : la fonction F(x) = ∫0
f(t).dt est intéressante. Elle est croissante, de limite I en +∞, C

en presque tout point, mais ayant des tangentes verticales en tous les nπ + π .
2
> with(plots):f:=t->exp(-t)/sqrt(abs(cos(t)));
A:=int(f(t),t=-Pi/2..Pi/2);B:=int(f(t),t=0..Pi/2);
C:=evalf(B+A/(exp(Pi)-1));
C := 1.481451160
> p:=plot(f(t),t=0..3*Pi,0..4,color=blue,numpoints=10000,thickness=2):
q:=plot(exp(-t),t=0..3*Pi):display({p,q});
> F:=x->int(f(t),t=0..x);plot([C,F(x)],x=0..3*Pi,thickness=2);

Exercice 8 : Lemme de Riemann-Lebesgue.


Soient f : I → K ou E une fonction intégrable, p : R → K une fonction réglée T-périodique.
T
Montrer que : limn→+∞ ∫I f(t).p(nt).dt = 1
T ∫
0
p(t).dt . ∫I f(t).dt .
π /2
Application : Montrer que la fonction de Bessel J0(x) = 2
π ∫
0
cos(x.sinθ).dθ tend vers 0 en ±∞.
[Poser sin θ = t].

20
Solution :

f(t² −1) 1 dt =
+∞ +∞ f(t)
Exercice 9 : Soit f : R → R continue et bornée. Montrer que ∫−∞ 2t 1+t² ∫−∞ 1+t²
dt .

Solution :

Exercice 10 : Formule de Schlömilch . Soient f : I = ]0, +∞[ → C réglée sur tout segment, a et c > 0.
On suppose que l’intégrale ∫I f((cx − a )2).dx converge.
x
Montrer que l’intégrale ∫I f(t2).dt converge, et que : ∫I f((cx − a )2).dx = 1 .∫I f(t2).dt .
x c

Solution : Schlömich est un mathématicien allemand, dont le nom est plus facile à prononcer quand
on a une pomme de terre chaude dans la bouche.

Exercice 11 : Soient p1, p2, …, pn des réels > 0, a1 < a2 < … < an.
n
pj
1) Etudier les variations de ϕ(x) = x − ∑ x −a
j =1 j
.

2) Soit y ∈ R. Combien l’équation ϕ(x) = y a-t-elle de solutions en x ? Quelle est leur somme ?
+∞ +∞
3) Soit f ∈ C(R, R) intégrable. Montrer que ∫ ∫−∞
f(ϕ(t)).dt =
−∞
f(t).dt .
+∞ p² +∞ p
4) Applications : Calculer ∫ exp(− t² − ).dt (p > 0), puis ∫ t −3/ 2 exp(−st − ).dt (s et p >0).
−∞2 2t ² 0 t

Solution :

Problème 12 : intégrale de fractions rationnelles.


P(x)
1) Soit F(x) = ∈ C(X) une fraction rationnelle sous forme réduite.
Q(x)
F est intégrable sur R si et seulement si F n’a pas de pôle réel, et deg Q ≥ deg P + 2.
+∞
P(x)
Et alors : ∫ .dx = 2iπ. ∑Resa(F) = −2iπ. ∑Resa(F) .
− ∞ Q(x)
a∈Π + a∈Π −

où Π (resp. Π+, Π−) est l’ensemble des pôles de F (resp. dans Im z > 0, dans Im z < 0) et Resa(F) est
le résidu de F en a (c’est-à-dire le coef. de 1 dans la décomposition en éléments simples de F).
z −a
2) Applications :
+∞ +∞
i) Calculer ∫−∞ xdx
4 +1
et ∫
−∞
dx .
x4 + x² +1
x2p
∫−∞ x2ndx+1 = n.sin(ππ /n) et π
+∞ +∞
ii) Montrer que ∫ −∞ x 2n +1
.dx =
2p +1
( p < n ).
n.sin( π)
2n
+∞ +∞
iii) Calculer ∫−∞ (x dx+1)n et F(a) =
2 ∫
−∞
dx
(x 2 +1)².(x² −2xcosa +1)
.

Solution :

21
3. Intégrales impropres et sommes de Riemann.

La théorie des sommes de Riemann s’étend-elle aux intégrales généralisées ? La réponse est
évidemment oui ( il y a des théorèmes ), mais le programme est muet sur ce sujet. Il faut donc se
débrouiller à la main.

n −1
Exercice 1 : Trouver limn→+∞ ∑
k =0
1 .
n²−k²

n −1 n −1
Solution : Sn = ∑
k =0
1
n²−k²
= 1
n ∑
k =0
1
1−(k /n)²
est une somme de Riemann de f(x) = 1
1− x²
associée à la subdivision ( 0, 1 , …, n−1 ). Elle tend vers ∫ f(x).dx = π2 …
1

n n 0

Mais il y a un problème ! Nous n’avons jamais démontré de théorème concernant la convergence


des sommes de Riemann dans le cas des intégrales impropres. Pour justifier rigoureusement ce
passage à la limite, on procèdera par un encadrement intégral basé sur la croissance de la fonction f
sur [0, 1[ :
1−1/ n n −1

1
1 +
n ∫0
f(x).dx ≤ Sn = 1
n k =0
1
1−(k /n)²
≤ ∫ f(x).dx .
0

Il reste à faire tendre n vers l’infini, à l’aide du lemme des gendarmes.

n −1
Exercice 2 : Trouver limn→+∞ ∑
k =0
1
n²+(−1)k.k²
.

Solution : [ Oral Mines PC 2012, RMS n° 697 ]


n −1
, où M = [ n−1 ] et N = [ n ] − 1.
M N
Sn = ∑
k =0
1
n²+(−1)k.k²
= ∑
p =0
1
n²+4p²
+ ∑
p =0
1
n²−(2p+1)² 2 2
La première somme se ramène à des sommes de Riemann classiques :
M M
∑ ∑
1/ 2
An =
p =0
1
n²+4p²
= M 1
n M p =0 p²
1 → 1
2 ∫
0
dx = 1 Argsh 1.
1+4x² 4
1+4

La seconde somme se ramène à des sommes de Riemann voisines de celles de l’exercice précédent.
dx = π .
N N
∑ ∑
1
Bn =
p =0
1
n²−(2p+1)²
= N 1
n N p =0
1
(2p+1)²
→ 1
2 ∫
0 1− x² 4
1−

n −1
Conclusion : Sn = ∑ 1 → 1 Argsh 1 + π .
k =0 n²+(−1) .k²
k 4 4

1
Exercice 3 : 1) Donner un exemple de fonction réglée f : ]0, 1] → R telle que ∫ f(x).dx converge
0
n
sans que la suite un = 1
n ∑ f(kn) converge.
k =1
1
2) Montrer que si f : ]0, 1] → R est réglée et bornée, alors l’intégrale I = ∫ f(x).dx converge, et la
0
n
suite un = 1
n ∑ f(kn) converge vers I quand n tend vers l’infini.
k =1

22
3) Soit f : ]0, 1[ → R monotone. Montrer les résultats suivants :
1 1
a) ∫ f(x).dx converge
0
⇒ un → ∫ f(x).dx ;
0
1
b) La convergence de (un ) n’implique pas celle de ∫ f(x).dx ( considérer f(x) = 1x − 1−1x
0
);
1
c) Si f est bornée en 0 ou en 1, la convergence de (un ) implique celle de ∫ f(x).dx .
0

Solution :

n −1
k )1/k = exp(− π² ) .
Exercice 4 : Montrer que limn→+∞ ∏(1 −
k =1 n 6

Solution : Passant au log, tout revient à montrer que :


n −1 n −1
∑ k1 ln(1− kn) → − π6² ,
k =1
ou encore que Sn = 1 n ln(1− k ) → − π² .
n ∑k
k =1 n 6
ln(1− x)
Or Sn est une somme de Riemann de la fonction f(x) = .
x
1 +∞ xn−1 +∞ 1 xn−1 +∞
Elle tend donc vers ∫ f(x).dx = − ∫ ∑ .dx = − ∑∫ .dx = − ∑ 1 = − π² .
1

n=1 n
0 n
n =1 n² 6
0 0
n =1
Il reste cependant deux points à préciser :
• La fonction f est négative, intégrable et décroissante sur [0, 1[, après prolongement en 0 (étude des
varaitions, ou développement en série); la convergence des sommes de Riemann reste encore vraie.
• Le théorème d’intégration terme à terme des séries s’applique.

Exercice 5 : Lemmes de pincement.


1) Soit f : ]0, +∞[ → R+ une fonction décroissante et intégrable. Montrer que, pour tout h > 0, la
+∞ +∞ +∞
série ∑ f(nh) converge, et que : lim h→0+ h ∑ f(nh) =
n =1 n =1
∫0
f(t).dt .

2) Soit f : [0, +∞[ → R une fonction réglée, décroissante sur [A, +∞[, et intégrable. Montrer que,
+∞ +∞ +∞
pour tout h > 0, la série ∑ f(nh) converge, et que : lim h→0+ h ∑ f(nh) =
n =1 n =1
∫0
f(t).dt .

+∞ N +∞
On pourra écrire h. ∑ f(nh) = h. ∑ f(nh) + h
n =1 n =1
∑ f(nh) , où N = [ Ah ] .
n= N +1

3) Soit f : ]0, +∞[ → E (Banach) une fonction réglée. On suppose qu’existe g : ]0, +∞[ → R
décroissante et intégrable, telle que (∀x > 0) || f(x) || ≤ g(x). Montrer que f est intégrable, que, pour
+∞ +∞ +∞
tout h > 0, la série ∑ f(nh) converge, et que : lim h→0+ h ∑ f(nh) =
n =1 n =1
∫ 0
f(t).dt .

Solution : 1) Rappelons qu’une fonction décroissante est réglée sur tout segment.
(N +1)h N +∞
f(x).dx ≤ h. ∑ f(nh) ≤
Nh
On a l’encadrement intégral : ∫h
n =1
∫0
f(x).dx ≤ ∫ 0
f(x).dx .
N +∞
La suite N → ∑ f(nh) est croissante majorée, donc elle converge, et la série ∑ f(nh) converge.
n =1 n =1
+∞ +∞ +∞
Fixons h > 0 et faisons tendre N vers l’infini. Il vient : ∫ f(x).dx ≤ h. ∑ f(nh) ≤ ∫ f(x).dx .
h 0
n =1

23
+∞ +∞
En vertu du lemme des gendarmes, h. ∑ f(nh) tend vers ∫
n =1
0
f(x).dx quand h tend vers 0+.
2) Soit f : [0, +∞[ → R une fonction réglée, décroissante sur [A, +∞[, et intégrable.
+∞
Pour tout h > 0, la série ∑ f(nh) converge, pour la même raison qu’en 1).
n =1
+∞ N +∞
Ecrivons h. ∑ f(nh) = h. ∑ f(nh) + h ∑ f(nh) , où N = [ Ah ] .
n =1 n =1 n= N +1
N
∑ f(nh) → ∫
A
D’une part, h. f(x).dx par un argument de sommes de Riemann.
0
n =1
+∞ +∞
D’autre part, h ∑ f(nh) → ∫
n= N +1
A
f(x).dx par encadrement comme ci-dessus.

Exercice 6 : Montrer que la fonction f : x → sin 1 est intégrable sur ]0, 1].
x
En déduire que la fonction f(x) = sin 1 si x ≠ 0 , f(0) = 0 , admet une primitive sur R.
x
Solution :
1) f est continue sur ]0, 1] et bornée, donc intégrable.
Remarques : i) Si on la prolonge en 0 par f(0) = a , f est Riemann-intégrable et non réglée.
1 +∞ sin y
ii) Le changement de variable y = 1 donne
x ∫ 0
sin x.dx =
x ∫
1 y²
.dy .
x
2) Considérons la fonction F(x) = ∫ 0
f(t).dt .

Elle est définie et continue sur R, dérivable et même C sur R*, car F’(x) = sin 1 .
x
Reste à montrer que F est dérivable en 0 et que F’(0) = 0.
Une intégration par parties donne :
x x x x
∫ ∫ −t².d(cos1t ) = − t .cos 1t | ∫ ∫
2 2
F(x) = f(t).dt = x
0 + 2 t.cos1.dt = − x .cos 1 + 2 t.cos1.dt .
0 0 0 t x 0 t
En toute rigueur, il faudrait intégrer sur [ε, x], puis faire tendre ε vers 0.
Cette formule est vraie pour tout x ≠ 0.
x
∫0 t
2
Elle reste vraie pour x = 0, car − x .cos 1 et 2 t.cos1.dt tendent vers 0 quand x tend vers 0.
x
F(x) x x
Par suite
x
= − x.cos 1 + 2 ∫ t.cos1.dt = − x.cos 1 + 2
x x 0 t x x ∫ g(t).dt , où g est continue sur R.
0

F(x)
Donc tend vers 0 et F’(0) = 0.
x
3) Voici une autre présentation : Les fonctions qui admettent une primitive sur R, c’est-à-dire les
fonctions dérivées, forment un espace vectoriel, image de D(R, R) par l’application D : y → y’. Cet
espace vectoriel contient, on le sait, les fonctions continues.
2 2
Or (x cos 1 )’ = 2x cos 1 − sin 1 . (x cos 1 )’ est une dérivée, 2x cos 1 est continue sur R, après
x x x x x
prolongement en 0, donc est une dérivée. Leur différence f est aussi une dérivée.
Cela montre que l’espace des fonctions primitivables contient strictement l’espace des fonctions
continues.

24
4. Intégration des relations de comparaison.

e 1 −t
Exercice 1 : Étudier F(x) = ∫x t .dt au V(0+).
Solution :
e−t
La fonction f(t) = est positive et non intégrable sur ]0, 1] car équivalente en 0+ à g(t) = 1 .
t t
On en déduit que F(x) tend vers +∞ quand x tend vers 0+ .
1
Le TIRC (IIc) donne aussitôt : F(x) ∼ ∫ dtt
x
= − ln x (*)
1 e −1
1 −t
mais on peut éviter d’appliquer le TIRC en écrivant : F(x) = ∫x dtt + ∫x t .dt = − ln x + H(x) ,
e −1 1 −t e−t −1
où H(x) = ∫x t . dt . La fonction h(t) =
t
est prolongeable par continuité en 0 (et même
e−t −1
développable en série entière), de sorte que l’intégrale ∫]0,1] .dt est faussement impropre.
t
e−t −1
Ainsi : F(x) = − ln x + c + o(1) , où c = ∫]0,1] .dt . Résultat plus précis que (*) !
t

Exercice 2 : Intégrale de Frullani.


e −e−bt
+ ∞ −at
Convergence et calcul de ∫0 t
.dt pour a et b > 0.

Solution : Bien que les fonctions intégrées ne se primitivent pas élémentairement, on peut calculer
cette intégrale impropre.
e−at −e−bt
La fonction f(t) = est continue sur ]0, +∞[ , prolongeable par continuité en 0 car
t
2
f(t) = 1 (1 − at − 1 + bt + O(t )) = b − a + O(t) → b − a, de sorte que l’intégrale est faussement
t
impropre en 0. Enfin, sur [1, +∞[ f est différence de deux fonctions intégrables.
+ ∞ −at
e + ∞ −bt
e
Attention, écrire : Ι = ∫0 t
.dt − ∫
0 t
.dt n’a pas de sens ... mais plutôt : I = limε→0+ I(ε) ,
e −e−bt
+ ∞ −at + ∞ −at
e + ∞ −bt
e + ∞ −u
e + ∞ −u
e e−u

où I(ε) = ∫ε t
.dt = ∫ε t
.dt − ∫ε t
.dt = ∫ε
a u
.du − ∫ε
b u
.du = ∫aε u .du
(chgts de var u = at et u = bt , puis Chasles) .
e−u
Or ∼ 1 > 0 au V(0+) . Donc, par un argument d’intégration de relations de comparaison, qui
u u

ne se déduit pas de (I, c), mais se montre comme (I, c), on a I(ε) ∼ ∫ du = ln b .
aε u a
bε bεe−t −1 bε
Une variante élémentaire consiste à écrire : I(ε) = ∫ε
a
dt +
t ∫aε t .dt = ln ba + ∫ ε g(t).dt ,
a

où g(t) est continue sur R. Donc ε → ∫ ε g(t).dt
a
est continue sur R, et tend vers 0 en 0. cqfd.

Remarque : On donnera dans la suite (§ 9) une autre méthode de calcul, en considérant l’intégrale
comme fonction de a.

25
1
x−1.dx .
Exercice 3 : Convergence et calcul de ∫
0 ln x

Solution : [ Oral X MP 2011, RMS n° 205 et Mines PC, n° 723 ]


1) S’assurer de la convergence de l’intégrale.
2) Le changement de variable s = − ln x, x = exp(−s) donne aussitôt :
e −e−2t
+∞ −t
I= ∫0 t .dt . C’est une Frullani, qui vaut ln 2.
1−ε 1−ε
3) On peut aussi noter que : I = limε→0+ ∫ x .dx − ∫ dx ,
0 ln x 0 lnx
2
puis faire le changement de variable y = x dans la première intégrale.
1−ε
I = limε→0+ ∫
(1−ε)²
− dx , et conclure par un argument d’intégration de relations de comparaison.
lnx

+ ∞ −t
e
Exercice 4 : 1) Etudier la fonction F(x) = ∫ x t²
.dt au V(0+) .
+∞ e−at −e−bt
2) Convergence et calcul de : ∫ 0
(
t
)².dt , pour a et b > 0.

e−at −e−bt 2
Solution : On montre aisément que f(t) = ( ) est intégrable sur ]0, +∞[. Développons :
t
+∞ e−at −e−bt
−(a +b)t
+∞ e−2at +∞ e +∞ e−2bt
J(ε) = ∫ ( )².dt = ∫ .dt − 2 ∫ .dt + ∫ .dt
ε t ε t² ε t² ε t²
+∞ e−u +∞ e−u +∞ e−u
= 2a ∫ .du − 2( a + b ) ∫ .du + 2b ∫ .du .
2aε u² (a +b)ε u² 2bε u²

en faisant les changements de variables respectifs u = 2at , u = (a + b)t et u = 2bt.


+ ∞ −u
e
Notons F(x) = ∫x u²
.du , de sorte que J(ε) = 2a.F(2aε) − 2(a + b) F((a + b)ε) + 2b.F(2bε) .
Il s’agit d’obtenir un développement asymptotique de F(x) au V(0+). Or écrivons
+1
1−u .du +e −1+u
+1 −u +∞ e−u
F(x) = ∫x u² ∫x u² . du + ∫1 u² .du .
+1 e−u −1+u +∞ e−u
= 1 + ln x + 1 + ∫ .du + ∫ .du + o(1) = 1 + ln x + C + o(1) .
x 0 u² 1 u² x
+1 e−u −1+u
car l’intégrale ∫ .du est faussement impropre en 0. Finalement :
0 u²
J(ε) = 2a ( 1 + ln ε + ln(2a) + C + o(1))
2aε
− 2(a + b) ( 1 + ln ε + ln(a + b) + C + o(1))
(a+b)ε
+ 2b( 1 + ln ε + ln(2b) + C + o(1)) = 2a ln(2a) – 2(a + b)ln(a + b) + 2b ln(2b) + o(1).
2bε
e −e−bt
+∞ −at +∞ e−at −e−bt
Si a, b > 0 , ∫
0 t
.dt = ln b et
a ∫0
(
t
)².dt = 2a ln(2a) – 2(a + b)ln(a + b) + 2b ln(2b).

Maple confirme :
> F:=x->int(exp(-t)/t^2,t=x..infinity);F(x);series(F(x),x=0,6);

⌠ ( −t ) ( −x )
 e −e + Ei( 1, x ) x
F := x → 
 t 2 dt −

 x
⌡x

26
1 1 1 1 4
x -1 + ( −1 + γ + ln( x ) ) − x + x 2 − x 3 + x + O( x 5 )
2 12 72 480
2x

x∫
Exercice 4 : Soit f : R+ → R+ continue intégrable. Pour x > 0, on pose g(x) = 1 f(t).dt .
x
+∞
Montrer que g est intégrable sur R*+ et calculer ∫0
g(x).dx .

Solution : [ Oral Mines 2009, RMS n° 505 ]


2

1
La fonction g est de classe C sur ]0, +∞[. Et g(x) = f(xu).du .
1
+∞ +∞ 2
1) Formellement, I = ∫
∫ (∫
0
g(x).dx =
0 1
f(xu).du).dx = ∫∫ f(xu).dudx , où D = ]0, +∞[×[1, 2].
D
2 +∞ 2 +∞ +∞
Intervertissons les intégrations : I = ∫ (∫ f(xu).dx).du = ∫ (1 ∫ f(t).dt).du = ln 2. ∫ f(t).dt .
1 0 u 1 0 0
+ −
Tout cela est légitime en vertu du théorème de Fubini. L’intégrabilité de g passe par f et f .
x F(2x)− F(x)
2) Autre solution : Notons F(x) = ∫ 0
f(t).dt ; alors g(x) =
x
.
+∞
La fonction F tend vers 0 en 0+, vers J = ∫ 0
f(t).dt en +∞ .
+∞ F(2x)−F(x)
I= ∫
0 x
.dx est alors une intégrale de Frullani.
+∞
Sa convergence vers ln 2. ∫0
f(t).dt se montre comme dans les exercices précédants.
A F(2x)−F(x) 2A F(x) A F(x) 2A F(x) 2ε F(x)
Elle passe par : ∫ε x
.dx = ∫ε
2 x
.dx − ∫ε x
.dx = ∫
A x
.dx − ∫ε x
.dx ,
et un argument d’intégration de relations de comparaison :
2ε F(x) 2A F(x)
∫ε x
.dx → 0 quand ε → 0 , et ∫A x
.dx → J.ln 2 quand A → +∞.

Exercice 5 : Trouver les parties principales au V(+∞) des fonctions :


x th(t) x x +∞
F(x) = ∫ 0 t(t +1)
.dt , F(x) = ∫0 (1+1t )t .dt , F(x) = ∫ e
ln(lnt).dt , F(x) = ∫ x
dt ,
t 4 +1

∫0 (1+t1²)−t²t 4 +1.dt , F(x) =


2x x x x exp(t)
F(x) = ∫ x
dt , F(x) =
t 4 +1 ∫1 tt.ln+1t.dt , F(x) = ∫1 t² +1
.dt .

Solution :
x th(t) x x x
♣ F(x) = ∫0 t(t +1)
.dt ∼ ∫ dtt
1
= ln x. ♦ F(x) = ∫ (1+1t ) .dt
0
t ∼ ∫ e.dt
0
= ex.

x +∞ +∞
♥ F(x) = ∫e
ln(lnt).dt ∼ x.ln ln x (I.P.P.) ♠ F(x) = ∫x
dt ∼
t 4 +1 ∫ x
dt = 1 .
t² x
2x 2x x x
♣ F(x) = ∫x
dt ∼
t 4 +1 ∫ x
dt = 1 .
t² 2x
♦ F(x) = ∫ tt.ln+1t.dt ∼ ∫ lnt.dt
1 1
∼ x.ln x.

27
+∞ +∞ dt
Exercice 6 : Nature de la série ∑u
n =1
n , où un = ∫ n t 7/2
+t 5/ 2
+ t 3 / 2 + t 1/ 2
. Equivalent du reste.

Solution :
La fonction intégrée f(t) est continue positive et intégrable sur [1, +∞[ car équivalente à 71/ 2 .
t
+∞ +∞
un existe donc. En vertu du T.I.R.C., un ∼ ∫
n
dt = 2 1 . Donc la série
t 7/ 2 5 n5/ 2 ∑u
n =1
n converge.
+∞ +∞
Enfin, en vertu du T.S.R.C., Rn = ∑u
k = n+1
k ∼ 2
5 ∑ k1
k = n+1
5/ 2
.
+∞ +∞
Enfin, un encadrement intégral laissé au lecteur donne ∑ k1
k = n+1
5/ 2
∼ ∫
n
dt = 2 1 .
t 5/ 2 3 n3/ 2
+∞
Finalement, Rn = ∑u
k = n+1
k ∼ 4 13/ 2 .
15 n

+∞ e−t
Exercice 7 : Etudier la fonction F(x) = ∫ x
t
.dt .

e−t
Solution : 1) Généralités. La fonction f(t) = est définie, continue et positive sur R*.
t
Elle est intégrable au V(+∞), car f(t) ≤ e−t pour t ≥ 1

Elle est intégrable au V(±0), car f(t) ∼ 1 au V((±0).


t
Par conséquent, le domaine de définition de F est R.
De plus, il est clair que F est décroissante sur R.
Sa limite en +∞ est nulle par définition d’un reste. Sa limite en −∞ est +∞, car f(t) → +∞ en −∞.
+ ∞ −t
e
Notons que F(0) = ∫0 t
.dt = Γ(½) = π .
2) Que dit Maple ?
> with(plots):f:=t->exp(-t)/sqrt(abs(t));
> plot(f(t),t=-3..5,0..8,thickness=2);

28
> F:=x->int(f(t),t=x..infinity);F(x);F(0);
 −I π erf( I −x ) x ≤ 0
π +  { 
 − π erf( x ) 0 < x 
> asympt(F(x),x,5);
( 3/2 ) ( 5/2 ) ( 7/2 )
1 11
+   −  
3 1 15 1
−  
x 2  x  4x 8 x
ex
> series(F(x),x=0,4);
Error, (in series/Heaviside) no series at 0
π
> plot(F(x),x=-2..1,thickness=2,color=blue,numpoints=1000);

3) Retour à F.
+∞ −t
e +∞ x
Si x ≥ 0, F(x) = ∫
x t
.dt = 2 ∫ e−u².du = π − 2 ∫ e−u².du : fonction parfaitement connue.
x 0

Elle a en 0+ un développement limité selon les puissances de x , ce qui explique la tangente


verticale en 0+. Elle a en +∞ un développement asymptotique à tous ordres obtenu par des
intégrations par parties répétées.

29
0 e−t −x
Si x < 0, écrivons F(x) = ∫
x −t
.dt + π = π +2 ∫
0
eu².du . Ici encore, fonction parfaitement

connue. Son étude en 0− permettrait d’expliquer sans peine la tangente verticale en 0.


Son étude en −∞ s’obtient par des intégrations par parties laissées au lecteur.

+∞ +∞
Exercice 8 : 1) Nature et calcul de ∫
0
dx.∫ e−t².dt .
x
2) Plus généralement, soit q : R+ → R+ une fonction continue, telle que x.q(x) soit intégrable.
+∞ +∞
Nature et calcul de : ∫ 0
dx.∫ q(t).dt .
x

Solution :
2
1) Nature. La fonction f(t) = exp(−t ) est continue, positive et intégrable sur R, car 0 ≤ f(t) ≤ 1 ,
1+t²
ou encore parce que 0 ≤ f(t) ≤ exp(−|t|) pour |t| ≥ 1.
+∞
La fonction F(x) = ∫x
exp(−t²).dt est elle-même continue, positive, et vérifie, pour tout x ≥ 1
+∞
0 ≤ F(x) ≤ ∫ x
exp(−t).dt = exp(−x) ; F est donc intégrable.
exp(−x²)
Variante : Une intégration par parties donne un équivalent de F(x) en +∞ : F(x) ∼ .
2x
2) Calcul. Intégrons par parties :
A A
∫0
F(x).dx = x.F(x) | 0A + ∫ x.exp(−x²).dx = A.F(A) −
0
1 exp(−x²) | 0A
2
tend vers 1 quand A → +∞ , car 0 ≤ A.F(A) ≤ A.exp(−A) pour A ≥ 1.
2
3) Autre solution : intégrales doubles impropres de fonctions positives
+∞ +∞

0
dx.∫ e−t².dt =
x ∫∫ e .dtdx où D = { (x, t) ; 0 ≤ x ≤ t }.
D
−t²

+∞ t +∞
= ∫ dt.∫ e .dx = ∫ t.exp(−t²).dt = − 1 exp(−t²) |
−t² +∞
0 = 1.
0 0 2 0 2
4) La généralisation est facile, et laissée au lecteur.

+∞ + ∞ −t
e
Exercice 9 : Nature et calcul de ∫ 0
(∫
x t
.dt).dx .

e−t
Solution : La fonction est intégrable sur toute demi-droite [x, +∞[, x > 0, mais pas sur ]0, 1].
t
+∞ e−t
La fonction F(x) = ∫
1
.dt est définie, positive, et de classe C sur ]0, +∞[.
x t
+∞
De plus, pour x ≥ 1, 0 ≤ F(x) ≤ ∫
x
e−t.dt = exp(−x).
Une intégration par partie fournirait même un équivalent de F en +∞.
+∞ e−t 1 −t
1.dt + 1e −1.dt + +∞ e .dt
e 1 −t −t
Si 0 < x ≤ 1, écrivons : F(x) = ∫x t . dt + ∫1 t . dt = ∫x t ∫x t ∫1 t
1 e−t −1 +∞ e−t
Donc F(x) = − ln x + ∫ .dt + ∫ .dt + o(1) quand x → 0.
0 t 1 t
Finalement, F est intégrable sur ]0, +∞[.
Pour calculer son intégrale, intégrons par parties :
A A A
∫ε F(x).dx = A.F(A) − ε.F(ε) − ∫ε x.F'(x).dx = A.F(A) − ε.F(ε) + ∫ε e −x.dx

30
= A.F(A) − exp(−A) + exp(ε) + ε.F(ε) → 1 quand ε → 0+ et A → +∞.
+∞ + ∞ −t
e
Conclusion : ∫0
(∫
x t
.dt).dx = 1.

5. Intégrales semi-convergentes.

La notion d’intégrale semi-convergente est au programme, mais celui-ci ne contient aucun théorème
+∞
les concernant. Il faut donc s’inspirer des méthodes utilisées lors de l’étude de ∫ 0
sint .dt .
t

+∞ +∞
Exercice 1 : Discuter la nature des intégrales ∫π sint .dt et
ta ∫ 0
sint .dt .
ta
+∞
Solution : ∫π sint .dt est • absolument convergente pour a > 1 ,
ta
• semi-convergente pour 0 < a ≤ 1 , • divergente pour a ≤ 0 .
Le 1 point est évident. Le 2ème s’établit par intégration par parties.
er

(n +1)π
Le 3ème s’établit en montrant que les tranches de Cauchy ∫π n
sint.dt ne tendent pas vers 0.
ta
π +∞
∫0 sin
ta
t .dt converge ssi a < 2 (règle de l’équivalent). Au final,

0
sint .dt converge ssi 0 < a < 2.
ta

+∞ +∞
Exercice 2 : Comparer les intégrales ∫
1
sint.dt et
t ∫1
[sint + sin ²t ].dt .
t t

cos(2t)
Solution : Les fonctions f(t) = sint et g(t) = sint + sin ²t = sint + 1 −
t t t t 2t 2t
sont équivalentes en +∞. Cependant, f est semi-intégrable, g ne l’est pas, car elle est somme de deux
fonctions semi-intégrables (par I.P.P.) et d’une fonction (celle du milieu) dont l’intégrale diverge.
Cela montre que la règle de l’équivalent ne s’applique pas pour les fonctions semi-intégrables.
+∞ (−1)n −1 +∞ (−1)n −1.n+ n
Ce contre-exemple est à rapprocher des deux séries : ∑
n =1 n
et ∑
n =1 n3 / 2
.

+∞ +∞
Exercice 3 : Étudier la nature des intégrales suivantes : ∫ 0
sin(e x).dx , ∫ −∞
sin(e x).dx ,

(−1)[1/ t]
1 +∞ +∞ +∞
∫0 t .dt (et calcul) , ∫0
sint .dt ,
t +cost ∫0
sin(t).sin(1).dt ,
t ∫
0
sin(t +1). dt
t t
+∞ +∞ +∞ +∞
∫ 0
sin(t + 1 ).dt ,
t ∫
1
tan(1).sin(t −1).dt ,
t ∫ 0
cos²t .dt (a > 0) ,
t a +cost ∫0
sint
cost +ln(1+t)
.dt

Solution :
X exp(X)
∫ ∫
x sinu .du .
1) Le changement de variable u = e donne sin(ex).dx =
0 1 u
+∞
Lorsque X tend vers +∞, ceci tend vers
u ∫
sinu .du . Il y a semi-intégrabilité.
1
Y exp(Y)
2) Le même changement de variable donne ∫ sin(ex).dx = ∫ sinu .du .
X exp(X) u

31
+∞
Lorsque X tend vers −∞, et Y vers +∞ indépendamment, ceci tend vers ∫ 0
sinu .du . Idem.
u
4) La fonction f(t) = sint est définie et continue sur [0, +∞[.
t +cost

sin(2t)
Ecrivons f(t) = sint ( 1 − cost + O( 1 ) ) = sint − + O( 31/ 2 ) .
t t t t 2t t
f est somme de deux fonctions semi-intégrables et d’une fonction intégrable.
+∞
Donc l’intégrale ∫
0
sint .dt converge.
t +cost
5) La fonction f(t) = sin t. sin 1 est continue sur ]0, +∞[.
t
Au V(0+), f est bornée, donc intégrable. Mieux même ! | f(t) | ≤ t ; il y a fausse impropreté.
Au V(+∞), écrivons : f(t) = sint + O( 13 ) ; f est somme d’une fonction semi-intégrable et d’une
t t
+∞
fonction intégrable. Conclusion : l’intégrale ∫0
sin(t).sin(1).dt converge.
t
6) La fonction f(t) = sin( t + 1 ) 1 est continue sur ]0, +∞[.
t t
Au V(0+), | f(t) | ≤ 1 , donc f est intégrable sur ]0, 1].
t
Au V(+∞), écrivons f(t) = 1 [ sin t.cos 1 + sin 1 .cos t ] = sint + O( 51/ 2 ) + O( 31/ 2 ).
t t t t t t
f est somme d’une fonction semi-intégrable et de deux fonctions intégrables.
+∞
Conclusion : l’intégrale ∫
sin(t +1). dt est convergente.
0 t t
7) La fonction f(t) = sin( t + 1 ) est continue sur ]0, +∞[.
t
Ecrivons f(t) = sin t.cos 1 + sin 1 .cos t.
t t
Au V(0+), f(t) = o(1) + O( 1 ) = O( 1 ), donc f est intégrable sur ]0, 1].
t t
Au V(+∞), f(t) = sint − sint + O( 31/ 2 ) + cost + O( 31/ 2 ).
t 2t t t t
F est somme de trois fonctions semi-intégrables et de deux fonctions intégrables.
+∞
Conclusion : l’intégrale ∫0
sin(t + 1 ).dt converge.
t

+∞
Exercice 4 : Nature de l’intégrale ∫
0
sin x.ln x.dx .
x²+1

Solution : [ Mines 1993, RMS n° 219 ]


C’est l’intégrale impropre en 0+ et en +∞ de la fonction f(x) = sin x.ln x .
x²+1
Au V(0+), f(x) ∼ x.ln x → 0 : intégrale faussement impropre.
Au V(+∞), la majoration | f(x) | ≤ .ln x ne conclut pas.
x²+1

32
Ecrivons 1 = 1 + O( 1 ) , donc f(x) = sin x.ln x + O( sin x.ln x ) = sin x.ln x + O( 1 ).
x²+1 x x3 x x3 x x2
g(x) = sin x.ln x est semi-intégrable sur [1, +∞[. En effet une intégration par parties donne :
x
cost(1−lnt)
cost.1−lnt.dt . Or
X X
∫1
sint.lnt.dt = − cost.lnt
t t
X
1 + ∫1 t² t2
= O( 31/ 2 ) …
t

+∞ +∞
Exercice 5 : Natures des intégrales : ∫0
x.sin(x3 − x).dx et ∫0
x.sin P(x).dx , P ∈ R[X] deg P ≥ 3.

Solution : [ Oral Centrale 1994, RMS n° 275 ].


3
La première intégrale est l’intégrale impropre en +∞ de la fonction continue f(x) = x.sin(x – x).
Cette fonction oscille fortement entre x et – x. Plusieurs approches sont possibles. Observons
3 ∞
d’abord que la fonction x → x – x est un C -difféomorphisme croissant de [1, +∞[ sur [0, +∞[.
1ère approche : intégration par parties.
X X
F(X) = ∫
+1
x.sin(x3− x).dx =
x .d(−cos(x3− x))
3x²−1 ∫
+1

= − X cos(X – X) + 1 − ∫ 3x²+1 .cos(x3− x).dx .


3 X

3X²−1 2 +1 (3x²−1)²

X cos(X3 – X) → 0 quand X → +∞, et l’intégrale X 3x²+1 .cos(x3− x).dx est absolument


Or −
3X²−1 ∫+1 (3x²−1)²
+∞
convergente. On en conclut que l’intégrale ∫
0
x.sin(x3 − x).dx converge.
Elle est d’ailleurs semi-convergente, car :
X X X 1−cos(2(x3− x))
∫ +1
x.sin(x3− x).dx ≥ ∫+1
x.sin²(x3− x).dx = ∫+1 x.
2
.dx

= X²−1 − 1
X

2 ∫
x.cos(2(x3− x)).dx .
4 +1
+∞
Or l’intégrale ∫
+1
x.cos(2(x3− x)).dx converge pour les mêmes raisons que ci-dessus.
X
Donc ∫+1
x.sin(x3− x).dx → +∞ quand X → +∞.
2ème approche : changement de variable.
3
Posons u = x – x. La bijection réciproque x = g(u) est implicite (à moins de recourir aux formules
+∞ +∞
de Cardan), mais enfin : ∫ +1
x.sin(x3− x).dx = ∫ 0
g(u).sin(u).g'(u).du .

Or g(u).g’(u) = x , et x ∼ u1/3 , puis, après calculs, x = g(u) = u1/3 + 1 u−1/3 + O(u−5/3).


3x²−1 3
−1/3 −5/3
On en déduit que g(u).g’(u) = 1 u + O(u ).
3
+∞ +∞
Ainsi, ∫ g(u).sin(u).g'(u).du = 1 ∫ sin u .du + +∞O( 1 ).du .
1 3 +1 u 1/ 3 ∫+1 u5/3
C’est la somme d’une intégrale semi-convergente et d’une intégrale absolument convergente.
Ces deux méthodes se généralisent à la seconde intégrale.

+∞
Exercice 6 : Calcul de I = sint.dt .
t ∫0

1) a) Montrer que la fonction f(x) = 1 − 1 est, une fois prolongée, de classe C1 sur ]−π, +π[.
x sin x

33
π /2
b) En déduire que limn→+∞ ∫
0
f(x).sin(nx).dx = 0 .
π /2 sin(nx)
2) Pour tout n ∈ N, on pose : Jn = ∫ 0 sin x
.dx .
Trouver une relation entre Jn et Jn−2 ( n ≥ 2 ). En déduire Jn .
+∞
3) Déduire de ce qui précède les valeurs de I et J = ∫ 0
sin ²t .dt .

Solution :
0) La fonction sin x est semi-intégrable, et non intégrable, sur R+ : question de cours.
x
1.a) La fonction f(x) = 1 − 1 est, une fois prolongée, de classe C1 sur ]−π, +π[.
x sin x

Tout d’abord, elle est C sur ]−π, 0[ ∪ ]0, +π[.
1
Le fait qu’elle soit de classe C sur ]−π, +π[ peut se montrer de deux façons.
1ère méthode : élémentaire. Elle consiste à faire un développement limité de f et de f’ en 0.
2
Il vient f(x) = − x + O(x ), ce qui montre que f(x) → 0 et f’(0) = − 1 .
6 6
Puis f’(x) = − 1 + cos x = − 1 + O(x), ce qui montre que f’(x) → f’(0) = − 1 quand x → 0 .
x² sin²x 6 6

2ème méthode : f est développable en série entière au V(0), donc C en 0.
En effet, f(x) = sin x− x . Après simplification par x , on trouve f(x) =
2 a(x)
, où a et b sont sommes
x.sin x b(x)
de séries entières de rayon infini, et b(0) = 1. Le théorème de division des séries entières s’applique
et montre que f est DSE(0).
π /2
b) Conséquence : limn →+∞ ∫0
f(x).sin(nx).dx = 0 .
On reconnaît le lemme de Riemann-Lebesgue. Il repose sur la continuité de f en 0. Mais ici, on peut
l’établir élémentairement, par intégration par parties et majoration en valeur absolue :
π /2 cos(nx) f(x).cos(nx) π / 2 π / 2 cos(nx)
In =
0 ∫ f(x).d(−
n
) =−
n 0 + ∫
0 n
.f'(x).dx = O( 1 ) .
n
π / 2 sin(nx)
2) Soit Jn = ∫ .dx . Pour n ≥ 2, il vient :
0 sin x
π / 2 2.cos((n−1)x).sin x
.dx = 2 ∫ cos((n−1)x).dx = 2 sin((n−1) π ) .
π /2
Jn − Jn−2 = ∫
0 sin x 0 n−1 2
Jn − Jn−2 = 0 si n est impair : la suite (J2k+1) est constante égale à π .
2
(−1) k +1 (−1)k +1
Jn − Jn−2 = 2 si n = 2k. Donc J2k = 2 ( 1 − 1 + 1 – … + ).
2k −1 3 5 2k −1
3) Conclusion. Ecrivons
π /2 sin(nx) π /2 nπ / 2 π /2
Jn = ∫
0 x
.dx − ∫
0
f(x).sin(nx).dx = ∫0
sint .dt −
t ∫
0
f(x).sin(nx).dx .
+∞
On déduit de c) que (Jn) tend vers ∫
0
sint .dt .
t
sint .dt = π . +∞
Faisons tendre n vers l’infini par valeurs impaires. Il vient
t 2 ∫0
+∞ (−1)k
Remarque : si l’on fait tendre n vers l’infini par valeurs paires, on trouve ∑ = π.
k =0 2k +1 4

34
+∞
Enfin, l’intégrale J = ∫
sin ²t .dt se calcule par parties :

0

.du = π .
+∞ +∞ +∞ sin(2t) +∞ sin(u)
J = ∫ sin ²t.d(−1) = ∫ 2sint.cost.dt = ∫ .dt = ∫
0 t 0 t 0 t 0 u 2

n π /2
Exercice 7 : Bis repetita. Calculer successivement : Sn = ∑sin((2k −1)x) ,
k =1
In = ∫
0
sin ²nx .dx ,
sin ²x
π /2 +∞ +∞
Jn = ∫
0
sin ²nx.cot g²x.dx , I = ∫ sin ²x.dx et J =
0 x² ∫ 0
sin x.dx .
x

Solution :

+∞ cos(at)−cos(bt)
Exercice 8 : Convergence et calcul de ∫0 t
.dt , a et b > 0 (Intégrale de Frullani).

Solution : Notons I(a, b) cette intégrale.


cos(at)−cos(bt)
Elle est faussement impropre en 0, car = O(t) au V(0+).
t
Elle est semi-convergente sur [1, +∞[, comme différence de deux intégrales semi-convergentes.
cos(at)−cos(bt) +∞
Pour calculer I(a, b), écrivons : I(a, b) = limε→0+
t ∫ε .dt .
+∞ cos(at)−cos(bt) +∞ cos(at) +∞ cos(bt) +∞ +∞
∫ε t
. dt = ∫ε t . dt − ∫ε t . dt = ∫aε cosx x.dx − ∫ ε cosx x.dx
b
bε bε bε bε
= ∫ cos x.dx = ∫ 1.dx + ∫ cos x−1.dx = ln b + ∫ f(x).dx ,
aε x aε x aε x a aε

où f(x) = cos x−1 est une fonction prolongeable par continuité en 0.


x

Par conséquent, ∫ ε f(x).dx → 0 quand ε → 0+. Au final :
a

+∞ cos(at)−cos(bt)

0 t
.dt = ln b .
a
e −e−bt
+∞ −at
Remarque : c’est la même technique que le calcul de ∫ 0 t
.dt avec une légère différence.

+∞ sin(xt)..sin(yt)
Exercice 9 : Convergence et calcul de I(x, y) = ∫0 t²
.dt .

Solution : [ Oral X 1984, RMS n° 126 ]


1) Intégrale faussement impropre en 0, absolument convergente sur [1, +∞[.
cos((x− y)t)−cos((x+ y)t)
+∞

2 ∫
2) Ecrivons I(x, y) = 1 .dt = 1 [ K(x + y) – K(x – y) ] ,
0 t² 2
+∞1−cos(xt) +∞
où K(x) = ∫ .dt = x ∫ 1−cosu.du si x > 0.
0 t² 0 u²
Une intégration par parties donne ∫ 1−cosu.du = ∫ sinu.du = π .
+∞ +∞

0 u² 0 u 2
De plus K(0) = 0. Par parité, K(x) = π |x|.
2
.dt = π [ | x + y | − | x – y | ].
+∞ sin(xt)..sin(yt)
Conclusion : I(x, y) = ∫
0 t² 4

35
+∞ sin3 x +∞ sin3 x
Exercice 10 : Convergence et calcul de A = ∫0 x2
.dx et de B = ∫0 x3
.dx .

Solution : [ Oral X 1993, RMS n° 79 ]


Les deux intégrales sont faussement impropres en 0, et absolument convergentes sur [1, +∞[.
+∞ 3.sin x−sin(3x) +∞ 3.sin x−sin(3x)
Ecrivons A = ∫0 4x 2
.dx = limε→0+ ∫ε 4x 2
.dx
+∞ +∞ sin(3x) +∞
= limε→0+ ∫ε 3.sin x.dx −
4x 2 ∫ε 4x 2
.dx = limε→0+ 3 F(ε) − 3 F(3ε) , où F(ε) =
4 4 ∫ε sin x.dx .
x2
Pour obtenir cette limite, étudions F au voisinage de 0+. Une IPP donne :
F(ε) = sinε + cos x.dx = sinε + 1cos x.dx + +∞ cos x.dx
+∞

ε ∫ε x ε ∫ε x ∫1 x
= sinε + ∫ dx + ∫ cos x−1.dx + ∫ cos x.dx = − ln ε + C + o(1),
1 1 +∞

ε ε x ε x 1 x
+∞
où C = 1 + ∫ cos x−1.dx + ∫ cos x.dx . On en déduit aussitôt :
1

0 x 1 x
+∞ sin 3 x
A = ∫ .dx = 3 ln 3.
0 x2 4
Variante par Frullani :
3 F(ε) − 3 F(3ε) = 3 sinε − sin(3ε) − 3 cos(3x)−cos x 3 +∞ cos x−cos(3x).dx ...
+∞

4 4 4 ε 4ε 4 ∫ε x
. dx →
4 ∫0 x
.dx = 3π .
+∞ sin x
3
Les mêmes techniques permettent de montrer que B = ∫
0 x3 8
+∞ sin p x
On peut calculer plus généralement les intégrales I(n, p) = ∫
0 xn
.dx lorsqu’elles convergent.

Remarque : Pour une raison que j’ignore, Maple se trompe dans le calcul de A.
> F:=(n,p,x)->int(sin(t)^p/t^n,t=x..infinity);J:=(n,p)-
>int(sin(t)^p/t^n,t=0..infinity); F(3,2,x);J(3,2);F(3,3,x);J(3,3);
1 −1 + cos( 2 x ) − 2 sin( 2 x ) x + 4 Ci( 2 x ) x 2 ∞

4 x2
1 −3 π x 2 + sin( 3 x ) + 3 cos( 3 x ) x + 9 Si( 3 x ) x 2 − 3 sin( x ) − 3 cos( x ) x − 3 Si( x ) x 2 3
− π
8 x2 8

Exercice 11 : Montrer qu’au voisinage de + ∞ :


1−α
+∞ +∞ +∞
sin ²t .dt ∼ x
∫x
sin ²t .dt ∼

1
2(α −1)xα −1
si α > 1 , ∫
x
sin ²t .dt ∼ 1 lnx ,
t 2 ∫ x tα 2(1−α)
si 0 < α < 1.

Solution :

+∞ +∞ +∞ +∞
Exercice 12 : Nature et calcul des intégrales ∫ ∫
0
(
x
sint .dt ).dx et
t ∫ ∫
0
(
x
cost .dt ).dx.
t

Solution :
Rappelons pour commencer que sin x est semi-intégrable sur R+, et cos x semi-intégrable sur toute
x x
demi-droite [x, +∞[, x > 0, mais non intégrable sur ]0, 1].

36
+∞
1) La fonction F(x) = ∫
x
sint .dt est définie et de classe C1 sur R . Il faut faire deux intégrations
t +

par partie pour obtenir un renseignement sur son comportement asymptotique en +∞.
+∞ +∞
F(x) = ∫ x
sint .dt = cos x + sin x − 2
t x x² ∫ x
sint .dt = cos x + O( 1 ) (*).
t3 x x²
Du coup, F est somme d’une fonction semi-intégrable et d’une fonction intégrable sur [1, +∞[.
+∞
Comme F a une limite en 0, l’intégrale ∫0
F(x).dx est faussement impropre en 0 et semi-conver-
gente en +∞. Pour la calculer, intégrons par parties derechef :
A A A
∫ε F(x).dx = A.F(A) − ε.F(ε) − ∫ε x.F'(x).dx = A.F(A) − ε.F(ε) + ∫ε sin x.dx
= A.F(A) − ε.F(ε) − cos A + cos ε → 1 quand A → +∞ et ε → 0+, en vertu de (*).
+∞
2) La fonction G(x) = ∫
x
cost .dt est définie et de classe C1 sur R . Il faut faire deux intégrations
t +*

par partie pour obtenir un renseignement sur son comportement asymptotique en +∞.
+∞ +∞
G(x) = ∫ x
cost .dt = − sin x + cos x − 2
t x x² ∫ x
cost .dt = − sin x + O( 1 ) (*).
t3 x x²
Du coup, G est somme d’une fonction semi-intégrable et d’une fonction intégrable sur [1, +∞[.
Attention, sur ]0, 1], la situation est moins simple qu’en 1). Ecrivons :
+∞ +∞
∫ dtt + ∫ costt−1.dt + ∫ ∫ costt−1.dt + ∫
1 1 1
G(x) = cost .dt = − ln x + cost .dt + o(1).
x x 1 t 0 1 t
Donc G(x), équivalente à − ln x, est intégrable sur ]0, 1].
Intégrons par parties derechef :
A A A
∫ε G(x).dx = A.G(A) − ε.G(ε) − ∫ε x.G'(x).dx = A.G(A) − ε.G(ε) + ∫ε cosx.dx
= A.G(A) + sin A − ε.G(ε) − sin ε → 0 quand A → +∞ et ε → 0+.
+∞ +∞ +∞ +∞
Conclusion : ∫ ∫
0
(
x
sint .dt ).dx = 1 et
t ∫ ∫
0
(
x
cost .dt ).dx = 0.
t
Avec Maple, qui reste perplexe (et se trompe) :
> F:=x->int(sin(t)/t,t=x..infinity);F(x);series(F(x),x=0,4);
asympt(F(x),x,4);int(F(x),x=0..infinity);
1 1
∞ π − x + x 3 + O( x 4 )
⌠ sin( t ) 1 2 18
F := x →  π − Si( x ) 0
 dt
+ O 4 
 2 cos( x ) sin( x ) 2 cos( x ) 1
 t + −
⌡x x 2 3 x 
x x  
> G:=x->int(cos(t)/t,t=x..infinity);G(x);series(G(x),x=0,4);
asympt(G(x),x,4);int(G(x),x=0..infinity);

⌠ cos( t ) 1
( −γ − ln( x ) ) + x 2 + O( x 4 )
G := x → 
 dt −Ci( x ) 4

 t
⌡x

+ O 4 
sin( x ) cos( x ) 2 sin( x ) 1
− + +
x 2 3 
x lim −x xCi( x ) + sin 
 x( x )
x→ ∞

x
Exercice 13 : 1) On considère la fonction F(x) = ∫ sin1t .dt .
0

a) Domaine de définition de F. Dérivabilité de F sur R*, en 0 ? Equivalent de F en +∞ ?


b) Variations et graphe de F ?

37
T
2) Soit f : R → R une fonction continue T-périodique (T > 0) ; on pose ω = 1
T ∫
0
f(t).dt .
x
a) Domaine de définition de F(x) = ∫
0
f(1).dt .
t
b) Dérivabilité de F sur R*, en 0 ? (Oraux ENS, 2002)

Solution :

+∞
Exercice 14 : Nature de l’intégrale ∫ 0
sin(sint).dt ?

Solution : La fonction f(t) = sin(sin t) est continue 2π-périodique.


Posons-nous plus généralement le problème : si f est continue 2π-périodique, à quelle condition
+∞
∫0
f(t).dt converge ? La réponse est : elle converge ssi f est nulle.

+∞
Exercice 15 : Soit f : [0, +∞[ → R uniformément continue. Montrer que l’intégrale ∫0
eif(t).dt
diverge.

Solution :

6. Espaces fonctionnels.

1 2 2
Exercice 1. On pose L (I) = { f ∈ C(I, R) ; f intégrable } et L (I) = { f ∈ C(I, R) ; f intégrable }
1 2
pour tout intervalle I de R. Etudier les relations d’inclusion entre L (I) et L (I) dans chacun des cas
suivants : I = [0, 1], I = ]0, 1], I = [0, +∞[.

Solution : [ Oral Mines MP 2010, RMS n° 481 ]


1 2
♦ Si I = [0, 1], il n’y a pas de problème L (I) = L (I) = C(I, R).
1 2
♦ Si I = ]0, 1], C(I, R) ⊃ L (I) ⊃ L (I), les inclusions étant strictes.
2
En effet de l’inégalité 2.| f | ≤ f + 1, on déduit que si f est de carré intégrable, f est intégrable.
Car la fonction 1 est intégrable sur I.
1
Les inclusions sont strictes, car 1a est toujours élément de C(I, R) ; elle est élément de L (I) ssi a <
x
2
1, de L (I) ssi a < ½.
1 2
♦ Si I = [1, +∞[, C(I, R) contient strictement L (I) et L (I), mais il n’y a aucune inclusion entre ces
deux espaces.
1
Les inclusions sont strictes, car 1a est toujours élément de C(I, R) ; elle est élément de L (I) ssi a >
x
2 2 1
1, de L (I) ssi a > ½. Donc 1 est élément de L (I) et non de L (I).
x
Reste à trouver une fonction f intégrable sur I, non de carré intégrable…
2
Ce n’est pas si facile, car si f tend vers 0 en l’infini, f ≤ | f | , donc f intégrable implique f de carré
intégrable… Il faut donc chercher une fonction ne tendant pas vers 0 en l’infini…
Considérons d’abord la fonction f telle que f(x) = n pour n − 13 ≤ x ≤ n + 13 , pour tout n ≥ 1, nulle
n n
ailleurs. Cette fonction est intégrable, mais non de carré intégrable. Si l’on veut une fonction

38
continue, considérer la fonction continue affine par morceaux g telle que g(n) = n, g(n ± 13 ) = 0,
n
g étant affine ailleurs.

+∞ +∞
Exercice 2 : Soit f ∈ C1(R+, R) telle que les intégrales ∫0
t².f²(t).dt et ∫
0
f'²(t).dt convergent.
+∞ +∞ +∞ +∞
Montrer que ∫0
f²(t).dt converge, et que : ∫0
f²(t).dt ≤ 2. ∫0
t².f²(t).dt . ∫
0
f'²(t).dt .
avec égalité ssi f est de la forme f(t) = A.exp(−λ.t2) (A ∈ R, λ > 0).

Solution : Inégalité de Weyl. Cf. mes problèmes d’intégration.

+
Exercice 3 : Soit f ∈ C2(R , R) telle que f et f '' soient de carré intégrable.
1) Montrer que f ' est de carré intégrable, puis que f.f' est intégrable.
2) Démontrer que limt→+∞ f(t) = 0 ; en est-il de même de f '(t) ?
+∞ +∞ +∞
3) Démontrer enfin que ∫0 ∫ f'²(t).dt ≤
0
f²(t).dt + ∫ 0
f''²(t).dt . Cas d’égalité ?
A
[ Indication : on pourra considérer F(A) = ∫ f 2 − f '2 + f ''2 − ( f + f ' + f '' )2 ]
0

Solution : Inégalité de Kolmogorov. Cf. mes problèmes d’intégration.

Exercice 4 : Soit f ∈ C2(R, R) telle que f et f'' soient de carré intégrable sur R.
Montrer que f' est de carré intégrable, et que ∫R f'2 ≤ ( ∫R f2 ).( ∫R f''2 ). Cas d’égalité ?

Solution :

7. Convergence monotone, convergence dominée.

exp(− x² ) +∞
Exercice 1 : Soit f : R → R continue à valeurs > 0. Pour tout n ∈ N*, soit an = ∫ n² dx.
−∞
f(x)+ 1

Montrer l’existence de an. Trouver une cns pour que la suite (an) converge.
Solution :

Exercice 2 : Soit (an) une suite de réels > 0 tendant vers 0. Pour tout x > 0, on pose
N(x) = card{ n ∈ N ; an ≥ x }.
Montrer que la fonction N est définie et monotone ; limites en 0+ et + ∞ ?
+∞ +∞ +∞
Montrer que ∑a
n =0
n converge ssi N est intégrable sur ]0, +∞[, et qu’alors ∑a = ∫
n =0
n
0
N(x).dx .

Solution : [Oral Mines PSI 2011, RMS n° 602 ]

Exercice 3 : calcul de l’intégrale de Gauss ∫R e−x² dx = π .


a) Montrer que e−x² est intégrable sur R.

39
π /2
π .
On rappelle l’équivalent de Wallis Wn = ∫ 0
sin nt.dt ∼
2n
b) Montrer que ∫R e−x² dx = limn→+∞ ∫R dx ; en déduire cette valeur.
(1+ x²)n
n
n
c) Montrer que ∫R e−x² dx = limn→+∞ ∫ (1 − x² )n.dx ; en déduire cette valeur.
− n n

Solution :
a) h(x) = e−x² est continue positive et intégrable car h(x) ≤ 1 , ou encore car h(x) ≤ e− x si |x| ≥ 1.
1+ x²
b) Considérons la suite de fonctions fn(x) = 1 .
(1+ x²)n
n
Ces fonctions sont toutes intégrables et convergent simplement vers la fonction h(x) = e−x² .
De plus 0 ≤ fn(x) ≤ f1(x). Le TCD s’applique.
On peut même montrer que fn(x) tend en décroissant vers h(x).
Le changement de variable x = n tan ϕ, ou plutôt ϕ = Arctan x ramène à des Wallis :
n
Jn = ∫R
π /2
π
dx = 2 n
(1+ x²)n

0
cos2(n−1)ϕ.dϕ ∼ 2 n
2(2n−2)
∼ π.
n
n
c) Considérons la suite de fonctions gn(x) = ( 1 − x² ) pour |x| ≤ n , gn(x) = 0 sinon.
n
Ces fonctions tendent simplement vers h(x). De plus 0 ≤ gn(x) ≤ h(x), qui est intégrable.
Le changement de variable x = n sin ϕ, ou plutôt ϕ = Arcsin x ramène aussi à des Wallis.
n
On peut montrer que gn(x) tend en croissant vers h(x).
d) La remarque qui tue… L’encadrement gn(x) ≤ h(x) ≤ fn(x) permet de conclure via les gendarmes,
sans utiliser le moindre théorème.

n
Exercice 4 : Limite de la suite In = ∫ (1+ nx) .e
0
n − 2x
.dx .

Solution : [ Oral X 1982 ]


+∞ n −2x
Ecrivons In = ∫0
fn (x).dx , où fn(x) = ( 1 + x ) .e
n
pour 0 ≤ x ≤ n, 0 pour x > n.
−x −x
fn(x) → e pour tout x, et 0 ≤ fn(x) ≤ e .
+∞
On conclut par convergence dominée que In → ∫0
e− x.dx = 1.

40
Remarque : en 1982, les programmes de taupe ne contenaient pas le théorème de convergence
dominée. Il fallait démontrer ce résultat à la main, en cassant en morceaux. A noter que l’on peut
calculer In, soit par binôme, soit par parties.

+∞
Exercice 5 : Soit f : R → R continue et bornée. Existence et limite de In = n
π ∫
−∞
f(x)e−n(x−a)².dx .

Solution : Il est facile de montrer que In existe pour n ≥ 1.


Il y a concentration d’aire au V(a), et l’on peut penser que la suite (In) se comporte comme
+∞ +∞
n
π ∫
−∞
f(a)e−n(x−a)².dx = f(a). n
π ∫
−∞
e−n(x−a)².dx = f(a), autrement dit que (In) tend vers f(a).
Cela peut se démontrer de plusieurs façons.
La plus simple est de faire le changement de variable u = (x – a) n .
+∞
Il vient : In = 1
π ∫−∞
f(a+ u )e−u².du .
n
2 2 2
Or fn(u) = f(a + u ).exp(−u ) → f(a).exp(−u ) et | fn(u) | ≤ || f ||∞. exp(−u ).
n
On conclut par convergence dominée.

+∞ −x n +∞ −x n
Exercice 6 : Limites et équivalents de Jn = ∫
0
e .dx et In = ∫+1
e .dx .

n
Solution : 1) Limites. Soit fn(x) = exp(−x ).
fn(x) est continue, positive, et intégrable sur [1, +∞[, car fn(x) ≤ f1(x) = exp(−x).
fn(x) → 1 si 0 ≤ x ≤ 1 , exp(−1) si x = 1 , 0 si x > 1.
De plus, il y a une majorante intégrable :
Sur [1, +∞[, on peut écrire : 0 ≤ fn(x) ≤ f1(x) = exp(−x).
Sur [0, +∞[, on peut écrire : 0 ≤ fn(x) ≤ ϕ(x), où ϕ(x) = 1 sur [0, 1[, exp(−x) sur ]1, ∞[.
Le théorème de convergence dominée montre que Jn → 1, et que In → 0 (d’ailleurs en décroissant).
n
2) Equivalents : Le changement de variable y = x donne resp. :
+∞ −1 1 +∞ 1 −1
Jn = 1
n ∫0
e− y.y n dy et In = 1
n ∫+1
e− y.y n dy .
+∞ 1 +∞
Or, par I.P.P. : Jn =
0∫ e− y.y dy → n

0
e− ydy = 1 par convergence dominée.
1
En effet : gn(y) = e y → e−y simplement, et il y a domination par 0 ≤ gn(y) ≤ φ(y), où
−y n

φ(y) = e−y sur [0, 1] , e−y y sur [1, +∞[.


+∞ 1 −1 +∞
Et In = ∫ +1
e− y.y n dy → ∫
+1
e− y.y−1dy derechef par convergence dominée.
1 −1
En effet : gn(y) = e− y y n → e−y y−1 simplement, et il y a domination par 0 ≤ gn(y) ≤ e−y .
+∞

n ∫
Conclusion : Jn → 1, In ∼ 1 e− y.y−1dy .
+1

Remarque : Jn = Γ(1 + 1 ) a un développement asymptotique à tous ordres, taylorien.


n
1 1 −1

∫e
−y
Jn − In = .y dy est une fonction gamma incomplète, qui a aussi un d.a. à tous ordres.
n
0

41
+∞
Exercice 6 : Montrer que la suite In = ∫0
dx
(1+ x)(1+ x )...(1+ x)
est définie pour n ≥ 2. Limite ?
2 n

Solution : La suite de fonctions fn(x) = 1 est décroissante, et tend simplement


(1+ x)(1+ x)...(1+ x )
2 n
vers la fonction f(0) = 1 , f(x) = 0 pour x > 0 (f étant discontinue, il n’y a pas convergence uniforme.)
n
En effet, si x > 0, ln fn(x) = − ∑ln(1+ kx) , somme partielle d’une série divergente à termes positifs,
k =1
n n
donc ln fn(x)→ −∞. Variante : − x Hn = − ∑ kx
k =1
≤ ln fn(x) ≤ − ∑(kx − 2xk²²) = − x Hn +
k =1
x² S .
2 n
Or la suite (Hn) tend vers +∞, tandis que (Sn) est convergente, donc bornée.
Comme 0 ≤ fn(x) ≤ f2(x) sur R+, la suite (In) tend vers 0 en vertu du théorème de convergence
dominée sur les intervalles quelconques.
Il s’agit de la convergence dominée du pauvre, puisqu’il y a convergence uniforme sur tout segment
inclus dans ]0, 1] ou ]0, +∞[.
Remarque : on peut obtenir un équivalent de In par cassage en 3 (cf. un problème d’intégration).

tn +∞
∑(−1) .u
1
Exercice 7 : Pour tout n, soit un = ∫01+t n .dt . a) Nature de la série n =0
n
n ?

b) Montrer que un = ln2 − π² + o( 1 ).


n 12n² n²

Solution : [ Oral Mines MP 2010, RMS n° 498 ]

+∞
Exercice 8 : Pour tout n, soit In = ∫
+1
dt . 1) Limite L de la suite (In), puis équivalent de In – L.
1+t n
2) Donner un développement asymptotique à 3 termes de In .

Solution :
1) Il est immédiat que In existe pour n ≥ 2, et que la suite (In) est décroissante.
fn(x) = 1 → ½ si x = 1, 0 si x > 1. De plus, 0 ≤ fn(x) ≤ 1 .
1+ xn 1+ x2
En vertu du théorème de convergence dominée, In → 0.
+∞ u1/n

n
2) Le changement de variable u = t donne In = 1 .du .
n +1 u(1+u)
u+∞ 1/ n +∞
Jn = ∫ .du → ∫ du = ln 2 par convergence dominée.
+1 u(1+u) +1 u(1+u)

u1/n 1 et 0 < u
1/ n
En effet → ≤ 1 pour n ≥ 2. Donc In ∼ ln2 .
u(1+u) u(1+u) u(1+u) u (1+u) n
3) Pour avoir le terme suivant, écrivons :
+∞ n(u1/ n −1) +∞
lnu .du = 1− ln s .ds
n.( Jn – ln 2 ) = ∫+1 u(1+u)
.du →
+1 u(1+u) ∫ ∫0 1+s
+∞ (−1)k −1
= ∫ −ln s.(1−s+s²+...)..ds = ∑ = π² .
1

0
k =1 k² 12

42
Justification laissée au lecteur par convergence dominée et intégration terme à terme des séries.
+∞
dt = ln2 + π² 1 + o( 1 ) quand n → +∞.
Conclusion : In = ∫+1 1+t n n 12 n² n²
Remarque : développement asymptotique à tous ordres.
+∞ elnu / n +∞ +∞ lnk u +∞ +∞ lnk u
In = 1
n ∫+1 u(1+u)
.du = 1
n ∫ ∑ k1!.n .u(1+u).du = ∑ k1! . n1 ∫
+1
k =0
k
k =0
k +1 +1 u(1+u)
.du .

Il s’agit d’un développement en série entière en 1/n qui fournit des développement asymptotiques à
tous ordres.

+∞
Exercice 9 : Limite et développement à deux termes de In = ∫
0
dx .
1+ xn

Solution : 1) Existence de la suite. Il est immédiat que In existe pour n ≥ 2.


2) Limite de la suite. fn(x) = 1 → 1 si 0 ≤ x < 1, ½ si x = 1, 0 si x > 1.
1+ xn
De plus, 0 ≤ fn(x) ≤ ϕ(x), où ϕ(x) = 1 si 0 ≤ x < 1, ½ si x = 1, 1 si x > 1.
1+ x2
En vertu du théorème de convergence dominée, In → 1.
3) Développement à deux termes.

4) Remarque finale. In est en fait une intégrale eulérienne, qui se rattache aux fonctions ∆, Β et Γ
via un changement de variable, la formule d’Euler et celle des compléments (cf. mon chapitre sur les
n
intégrales eulériennes, § 7). Le changement de variable u = t et ces formules donne en effet :
In = 1.∆(1 ,1) = 1.B(1− 1 , 1 ) = 1.Γ(1− 1 ).Γ(1) = π .
n n n nn n n n n.sin π
n
On peut alors obtenir un développement asymptotique à tous ordres.
> J:=n->Pi/(n*sin(Pi/n));asympt(J(n),n,15);
1 2 7 4 31 127 73 1414477
π π π6 π8 π 10 π 12
6 360 15120 604800 3421440 653837184000
1+ 2 + + + + +
n n4 n6 n8 n 10 n 12
8191
π 14
+ O 16 
37362124800 1
+ 14 n 
n  
+∞ t −[t] 1
Exercice 10 : Montrer que : γ = 1 − ∫ .dt = 1 − ∫ (1−[1]).dt . En déduire 0 < γ < 1.
+1 t² 0 t t

Solution :
Montrons la convergence de chacune des deux intégrales. Les deux fonctions sont réglées sur tout
t −[t] 1
segment, et 0 ≤ f(t) = ≤ et 0 ≤ g(t) = 1 − [ 1 ] ≤ 1.
t² t² t t
Le changement de variable u = 1/t ramenant une intégrale à l’autre, il suffit de montrer une formule.
1

∫ (1−[1]).dt = ln( 1 + 1 ) − 1 , de sorte que :


n
Par exemple 1
n +1
t t n n+1
1
∫ 1
N +1
(1−[1]).dt = ln(N + 1) − 1 − 1 − … − 1 → 1 − γ quand N → +∞.
t t 2 3 N +1

43
1
On en déduit 0 < γ < 1, car chacune des intégrales ∫ (1−[1]).dt est > 0 et < 1 .
n
1
n +1
t t n(n+1)

∑ 1k .
n 1
Exercice 11 : Montrer que ∀n ≥ 1 ∫ 1
1 (1− x)n.dx −
x n ∫0 1x[1−(1− nx)n].dx = ln n − k =1
11−e −x + ∞ −x
e
En déduire γ = ∫
0 x
.dx − ∫1 x
.dx .

Solution : Notons I(n) le premier membre.


n 1 n n
1[(1− x )n −1].dx − 11 [1−(1− x)n].dx
I(n) =∫1 1 (1− x)n.dx −
x n ∫0 1x[1−(1− nx)n].dx =
∫1 x n ∫0 x n ∫1 dxx +
n 1 11−v n
= ln n − ∫ 1[1−(1− x)n].dx = ln n − ∫ 1[1−(1−u)n].du = ln n − ∫ .dv
0 x n 0u 0 1−v
n
= ln n − ∫ (1+v+...+vn−1).dv = ln n − ∑ 1 .
1

k =1 k
0

Il reste à conclure par convergence dominée.

+∞ n
Exercice 12 : Montrer que ∫
0
e− x( 1 − x − 1).dx = γ , où γ = limn
1−e x ∑ 1k
k =1
− ln n .

Solution :
−x
1) Convergence. La fonction f(x) = e ( 1 − x − 1 ) est continue sur ]0, +∞[.
1−e x
Elle est prolongeable par continuité en 0, car limx→0 f(x) = 1 ; intégrale faussement impropre en 0.
2
−x
f(x) ∼ e en +∞ ; donc f est intégrable sur [1, +∞[, et finalement sur ]0, +∞[. Soit I sont intégrale.
n−1
2) Calcul. L’idée est d’utiliser la formule 1 = 1 + u + u2 + … + un−2 + u .
1−u 1−u
+∞ e−(n−1)x 1
Ecrivons I = ∫ 0
e− x(1+e− x +...+e−(n−2)x +
1−e− x x
− ).dx =

+∞ e −e−x
+∞ −nx +∞
= ∫
0
e− x(1+e− x +...+e−(n−2)x).dx + ∫
0 x
.dx + ∫
0
e−nx( 1 − x − 1).dx .
1−e x
La première intégrale vaut 1 + 1 + … + 1 .
2 n−1
La deuxième est une intégrale de Frullani ; elle vaut – ln n (cf. exercice antérieur).
+∞
La troisième s’écrit Kn = ∫ 0
e−nx.b(x).dx , où b(x) = 1 − 1 est bornée sur ]0, +∞[, car elle tend
1−e− x x
vers ½ en 0 et 0 en +∞. Or Kn tend vers 0,
−nx −nx −x
soit en vertu du théorème de convergence dominée : e .b(x) → 0 et | e .b(x) | ≤ e .| b(x) |,
+∞ +∞
soit par majoration élémentaire : | Kn | ≤ ∫0
e−nx.b(x).dx ≤ B ∫ e−nxdx = B .
0 n
Références :
Polya-Szegö, Problems and theorems in analysis, t. 1, n° 32 p. 54
Dieudonné, Calcul infinitésimal, n° 13, p. 163 (qui propose une autre solution)
Tuloup, Cours d’analyse
Centrale 1976 RMS 491 p. 206, etc.

44
+∞ n
Exercice 13 : Montrer que ∫
0
e− x ln x.dx = − γ , où γ = limn ∑ 1k − ln n .
k =1

1−e
+1 −x +∞ −xe
En déduire ∫
0 x
.dx − ∫
1 x
.dx = γ .

Solution :
−x
1) Convergence. La fonction f(x) = e .ln x est continue sur ]0, +∞[.
Au V(0+), f(x) ∼ ln x, qui est intégrable sur ]0, 1].
−x/2
Au V(+∞), 0 ≤ f(x) ≤ e ou 1/x² selon les goûts.
Conclusion : f est intégrable sur ]0, +∞[. Soit I sont intgrale.
+∞

1
Remarque : I = Γ’(1), car la fonction Γ(x) = t x−1e−t.dt , définie sur ]0, +∞[, est de classe C et a
0
+∞
pour dérivée Γ’(x) = ∫ 0
lnt.t x−1e−t.dt .
n
2) Calcul. Une idée est de monter que I = limn→+∞ ∫ (1− nx) .ln x.dx .
0
n

n +∞
Cela se montre par convergence dominée. Ecrivons : In = ∫ (1− nx) .ln x.dx = ∫
0
n
0
fn (x).dx ,
n
où fn(x) = (1 − x ) .ln x pour 0 < x ≤ n , 0 pour x ≥ n.
n
Or fn(x) → exp(−x).ln x et vérifie | fn(x) | ≤ exp(−x).| ln x |.
Reste à calculer l’intégrale In, ce qui n’est pas si facile ! Tout d’abord, par chgt de variable :
1 1 1
In = ∫ (1− y) .ln(yn).ndy
0
n

= n.ln n. (1− y)n.dy + n. (1− y)n.ln(y).dy
0 ∫
0

1−(1− y)n+1 1
= n .ln n + n. ∫ ln(y).d( )
n+1 0 n+1
Il y a nécessité d’introduire la constante 1 dans la forme différentielle sans quoi il y a problème !
n+1
1 1−(1− y) n +1 11−(1− y)n +1
Jn = ∫ ln(y).d( ) =− 1 ∫ .dy ( la partie intégrée étant nulle ).
0 n+1 n+1 0 y
11− xn +1 1
=− 1 ∫ .dx = − 1 ∫ (1+ x+ x²+...+ xn).dx = − 1 Hn+1 .
n+1 1− x
0 n+1 0 n+1
En conclusion, In = n [ ln n − Hn+1 ] , et l’on conclut aussitôt.
n+1
3) Conséquence. Intégrons par parties, mais attention ! il faut casser en deux.
+∞ +1 +∞ +1 +∞
∫0
e− x ln x.dx = ∫0
e− x ln x.dx + ∫1
e− x ln x.dx = ∫
0
ln x.d(1−e− x) + ∫
1
ln x.d(−e− x)
1−e−x
+1 +∞ −xe
=− ∫
0 x
.dx + ∫ 1 x
.dx . Voilà qui nous ramène à un exercice antérieur !

4) L’expression intégrale obtenue permet un calcul approché de la constante d’Euler.

Exercice 14 : Étudier les suites de fonctions suivantes, et leurs intégrales :


fn(x) = na.x.( 1 − x )n sur [0, 1] et gn(x) = na.x.exp(−n.x) sur R+ .
Le théorème de convergence dominée s’applique-t-il ?

Solution :

45
Exercice 15 : On considère la suite de fonctions fn : x ∈ [0, 1] → sin(nx) .
Montrer qu’aucune suite extraite (fn(k)) ne converge simplement sur [0, 1] vers 0.
[ Indication : On pourra considérer la suite ∫[0,1] fn(k)(x) .dx . ]
2

1
1 − sin(2n) → 1 .
Solution : ∫ sin²(nx).dx =
0 2 4n 2
1
Si une suite extraite (fn(k)) convergeait simplement sur [0, 1] vers 0, la suite ∫ sin²(n x).dx tendrait
0
k

vers 0 par convergence dominée. Or on vient de montrer le contraire.

Exercice 16 : Soit I un intervalle de R, (fn) une suite de fonctions continues par morceaux de I dans
R, convergeant simplement vers f continue par morceaux.
1) On supppose (∀n) fn ≥ 0. Montrer limn→+∞ ∫I | fn − f | = 0 ⇔ limn→+∞ ∫I fn = ∫I f.
2) Contre-exemple si on enlève l’hypothèse (∀n) fn ≥ 0 ?

Solution :

8. Intégration terme à terme des séries.

+∞ (−1)n −1 +∞

∑ ∑ n1 .
1 1
Exercice 1 : Montrer que : ∫ x x.dx =
0
n =1 nn
et que ∫ x− x.dx =
0
n =1
n

Calculer ces intégrales à 10−6 près.

Solution : Il s’agit des deux « series mirabili » découvertes par Johann Bernoulli en 1697.
Nous traiterons seulement la première intégrale.
1 +∞ xn lnn x +∞

∫ exp(x.ln x).dx = ∫ ∑ ∑ n1! ∫ x .ln x.dx .


1 1 1
1) Formellement ∫ x .dx
0
x =
0 0
n=0 n!
.dx =
n =0
0
n n

1
Reste à calculer les intégrales In = ∫ x .ln x.dx . Une première tentative d’intégration par parties
0
n n

1
échoue, car elle ne ramène pas In à In−1 , mais In à ∫ x .ln x.dx . n n −1
0
1
Il faut donc généraliser la question en considérant In(a) = ∫ x .ln x.dx , pour a > 0. a n
0

−(n−1) (−1)nn!
Alors une ipp donne In(a) = −n In−1(a) ; donc In(a) = −n … −1 I0(a) = .
a+1 n +1 a+1 a+1 a+1 (a+1)
La situation rencontrée obéit au principe “ more may be less ”. On la rencontre aussi lors du calcul
1 1
∫ x .(1−x) .dx ∫ x .(1−x) .dx .
n n n p
de In = : il est plus facile de calculer In,p =
0 0
1 (−1)nn!
Revenons z-a nos moutons ! Finalement, ∫ x .ln x.dx = In(n) =
0
n n
(n+1)n+1
, puis :

+∞
1 (−1) n! =
n +∞ (−1)n +∞ (−1)n −1
∑ ∑ ∑
1
∫ x x.dx =
0
n =0 n! (n+1)
n +1
n =0 (n+1)
n +1
=
n =1 nn
.

2) Reste à justifier l’interversion des limites. Ici, on a l’embarras du choix :


1ère méthode : la fonction f(x) = exp(x.ln x) est continue sur [0, 1] après prolongement par 1 en 0.
+∞(xln x)n
Elle est somme de la série ∑
n =0 n!
.

46
Cette série est normalement convergente sur [0, 1], car la fonction, x.ln x est bornée sur [0, 1],
comme le montrent ses variations : |x. ln x| ≤ 1 .
e
2 ème
méthode : on peut recourir au théorème d’intégration ∑∫ ]0,1]
un < +∞, qui s’applique puisque la
+∞
série ∑ n1
n =1
n
converge.

3) Calcul numérique très facile via le critère des séries alternées.


Inutile de dire que la série converge vers l’intégrale bien plus vite
que les sommes de Riemann. Historiquement d’ailleurs, au
XVIIème siècle, on calculait souvent les intégrales par dévelop-
pement en série de la fonction, avant de disposer de théorèmes
rigoureux.
> J:=int(x^x,x=0..1);evalf(J);
plot([x^x,0],x=0..1,thickness=2);
.7834305107
> S:=n->sum((-1)^(k-1)/k^k,k=1..n);
for n from 1 to 10 do evalf(S(n));od;
1. .7500000000 .7870370370 .7831307870 .7834507870
.7834293536 .7834305678 .7834305082 .7834305108 .7834305107
+∞
= π² .
+∞
Exercice 2 : Montrer que : ∫0
x .dx =
e x −1 ∑ n1²
n =1 6

Solution : 1) Formellement
+∞ +∞ x.e−x +∞ +∞ +∞ +∞

∫ x .dx =
∫ .dx = ∫ x.e− x. ∑e−nx .dx = ∫ ∑ x.e − nx .dx
0 e x −1 0 1−e− x 0
n =0
0
n =1
+∞ +∞ +∞
= ∑ ∫
n =1
0
x.e−nx.dx = ∑ n1²
n =1
, après un bref calcul.
+∞ +∞
∫ 0
x.e−nx.dx = 1
n² ∫
0
u.e−u.du = 1 ( chgt de var u = nx, puis intégration par parties ).

2) Or un(x) = x.e−nx est une fonction continue et intégrable sur R+ .
+∞ x.e−x
La série ∑un(x) converge simplement sur R*+ , et a pour somme la fonction f(x) =
n =1
1−e− x
.
+∞ +∞ +∞ +∞
Enfin : ∑∫ u (x).dx = ∑∫
n =1
I
n
n =1
0
xe−nx.dx = ∑ n1²
n =1
< +∞.

Le théorème d’intégration terme à terme des séries s’applique : f est intégrable et...
+∞
NB : 1) Par continuité f(0) = 1, tandis que ∑u (0) = 0. D’où la nécessité de se placer sur R*+ .
n =1
n

2) On peut éviter tout recours à un théorème, en raisonnant élémentairement ainsi :


f(x) est continue en 0, et intégrable sur ]0, +∞[ car 0 < f(x) < exp(−x/2) pour x assez grand.
n x.e−nx
L’identité exacte : (∀x > 0) f(x) = ∑ x.e
k =1
− kx +
ex −1
s’intègre terme à terme par linéarité :

+∞ n +∞ +∞ x.e−nx n +∞

0
x .dx =
e x −1 ∑∫
k =1
0
xe−kx.dx + ∫
0 e x −1
.dx = ∑ k1² + ∫
k =1
0
f(x).e− nx.dx .

47
+∞ +∞
Or 0 ≤ ∫0
f(x).e− nx.dx ≤ || f ||∞ ∫ e−nx.dx = 1 || f ||∞ → 0 . Conclure en faisant tendre n vers +∞.
0 n

+∞ xα
Exercice 3 : Montrer que ∫0 e x −1
.dx = Γ(α + 1).ζ(α + 1) pour α > 0.

Solution :
Cet exercice généralise le précédent. La solution est identique. A noter que la formule obtenue relie
les fonctions ζ et Γ. D’autres liens entre ces deux fonctions permettent de prolonger la fonction ζ.
Les formules pour α = 3 et 5 servent en physique pour le rayonnement du corps noir (Boltzmann, loi
de Stefan, cf. CCP 2007). Ah ! que j’aime voir rayonner les corps dans le noir…

+∞
Exercice 4 : Soit un = 1!+2!+...+ n! . Montrer que la série ∑u converge, et que :
(n+ 2)!
n
n =1
+∞
e−2+ ∑u
n =1
n = − ∫(0,1) et.ln t.dt .

Solution :

+∞
Exercice 5 : Montrer ∫(0,1) x².ln x .dx = ∑ (2n1+1)² = π² − 1.
x²−1 n =1 8
+∞
∫(0,1) ln(x).ln(1 − x).dx = ∑ n(n1+1)² = 2 − π² .
n =1 6
ln x.ln(1− x) +∞
∫(0,1)
x
.dx = ∑ n1
n =1
3
(constante d’Apéry ζ(3)).

ln(x²).ln(1− x²) +∞
∫(0,1)

.dx = ∑ n(2n2−1)²
n =1
.
+∞ (−1)n
∫(0,1) − ln x .dx = ∫(0,1) Arctan x .dx =
1+ x² x ∑ (2n+1)²
n =0
(constante de Catalan)

Solution :
+∞
1) Formellement, écrivons ∫(0,1) x².ln x .dx = − ∫(0,1) x².ln x .dx = − ∫(0,1) ln x x2n .dx
x²−1 1− x² n=1

+∞ +∞
= − ∑∫ x2n.ln x.dx = ∑ (2n1+1)² = π² − 1.
n =1
(0,1)
n =1 8
+∞ +∞ +∞ +∞
La dernière égalité découle de π² = ∑ n1² = ∑ (21n)² + ∑ (2n1+1)² = 1 π² + ∑ (2n1+1)² .
6 n =1 n =1 n =0 4 6 n =0

L’avant dernière égalité découle de ∫ x2n ln x.dx = 1 , qui se montre par parties.
(0,1) (2n+1)²
Le nœud de l’affaire tient dans l’application du théorème d’intégration terme à terme des séries à
+∞
∑u (x) , où
2n
n un(x) = x .ln x sur I = ]0, 1[. Chacune de ces fonctions est intégrable sur I, car
n =1
+∞
prolongeable par continuité en 0 et en 1, ∑u (x) =
n =1
n
x².ln x , et enfin
1− x²
+∞ +∞
∑∫
n =1
I
un(x).dx = ∑ (2n1+1)²
n =1
< +∞.

48
+∞ te−3t
Autre approche : le chgt de variable t = − ln x donne ∫(0,1) x².ln x .dx = ∫ .dt
x²−1 0 1−e−2t
Il reste à développer la fonction en série géométrique, comme dans l’exercice 2.
3) Formellement,
ln x.ln(1− x) +∞ (−1)n−1xn +∞ (−1)n−1 +∞
∫(0,1)
x
.dx = ∫(0,1) ln x
x ∑
n =1 n
.dx = ∑n =1 n ∫ (0,1)
xn−1ln x.dx = ∑ n1
n =1
3
.

Le théorème d’intégration terme à terme des séries s’applique sans difficulté.

Exercice 6 : Calculer, c’est-à-dire développer en série :


∫(0,+∞) −ln(th x).dx , ∫(0,+∞) x .dx
2.shx
ch(xt) sh(xt)
F(x) = ∫(0,+∞) .dt , G(x) = ∫(0,+∞) .dt ,
cht sht
A = ∫(0,1) ln x .dx , B = ∫(0,1) ln x .dx , C = ∫(0,1) ln x .dx .
1− x² 1− x 1+ x

Solution :

Exercice 7 : Constantes de Nielsen-Ramanujan.


2ln k x
Montrer que les intégrales ak = ∫1 x−1 .dx ( k ≥ 1 ) convergent, et que a1 = ζ(2)/2 , a2 = ζ(3)/4.
Solution :
Remarque : Pour k ≥ 3, les formules sont moins simples, et font intervenir ζ et les polylogarithmes.
(cf. Internet, Mathsoft)

+∞ +∞
∑ x²+x n²
sin(xt)
Exercice 8 : Montrer l’identité : (∀x ∈ R) ∫0 et −1
.dt =
n =1
.

Solution : 1) Formellement,
+∞ +∞ sin(xt).e−t +∞ +∞ +∞ +∞ +∞

∫ ∑sin(xt).e ∑∫ ∑ x²+x n² .
sin(xt)

0 et −1
.dt = ∫0 1−e−t
.dt =
0
n =1
−nt
.dt =
n =1
0
sin(xt).e−nt.dt =
n =1
+∞ +∞ exp[(ix−n)t]
En effet, ∫0
sin(xt).e−nt.dt = Im ∫ 0
e(ix−n).t.dt = Im
ix−n
+∞
0 = Im 1 = x .
n−ix n²+ x²
+∞
2) Justifcation : Appliquons le théorème d’intégration terme à terme des séries à ∑u (x) , où
n =1
n

un(x) = sin(xt).exp(−nt). Chacune de ces fonctions est continue et intégrable sur ]0, +∞[.
+∞ +∞ +∞ +∞ x
De plus, ∑∫
n =1
0
un(t).dt < +∞. En effet, | sin u | ≤ | u |, donc ∫0
un(t).dt ≤ ∫
0
x.t.exp(−nt).dt =

.

Exercice 9 : On admet ∫R exp(−t2).dt = π . Calculer les intégrales ∫R+ t2n.exp(−t2).dt .


En déduire que, pour tout z ∈ C, la fonction t → cos(z.t).exp(−t2) est intégrable sur R+ et calculer
son intégrale.

Solution :

Exercice 10 : Soit F(a) = ∫(0,1) ln(x).ln(1 − xa).dx , a > 0. Existence de l’intégrale ?

49
Développer F(a) en série. Limite de F(a) en 0 ? Limite et équivalent de F(a) en +∞ ?

Solution :

+∞ (−1)n

1
Exercice 11 : Montrer l’identité : ∫
0
dt =
1+tα n =0 nα +1
pour tout α > 0 .

Solution : Nous sommes ici sur le segment [0, 1].


1 +∞ +∞ +∞ (−1)n
∫ ∑(−1) ∑ ∫ (−1) ∑
1 1
Formellement : ∫01+dttα = 0
n =0
n tnα dt =
n =0
0
n tnα dt =
n =0 nα +1
.

Mais aucun des deux théorèmes d’intégration terme à terme des séries (celui sur les segments, celui
sur les intégrales impropres) ne s’applique :
t (n +1)α
• le reste Rn(t) = (−1)n+1 , ne tend pas uniformément vers 0, car sup[0,1] | Rn(t) | = 1 .
1+tα 2
+∞
• par ailleurs ∑n≥0 ∫I |un(x)|.dx = ∑ nα1+1 = +∞ .
n =0
Il reste à montrer les choses élémentairement, i.e. :
t (n +1)α n (−1)k 1t
(n +1)α


1 1
∫01+dttα = ∫ [1 − tα + ... + (−1)n.tnα + (−1)n+1
0 1+tα
].dt =
k =0 kα +1
+ (−1) n+1
∫0 1+tα .dt .
1 t (n +1)α 1
∫0 1+tα .dt ≤ ∫0t .dt = (n+11)α +1 . cqfd.
(n +1)α
Or 0 ≤

On peut aussi procéder par encadrement et gendarmes, 1 étant compris entre deux sommes
tα +1
partielles consécutives de 1 − tα + ... + (−1)n.tnα .

Exercice 12 : Soit (λn) une suite croissante de réels > 0, tendant vers +∞.
+ ∞ +∞ +∞ (−1)n
Montrer que : ∫
0
∑(−1)n.e−λn.x.dx =
n =0

n =0 λn
.
+∞ (−1)n +∞ (−1)n
En déduire les sommes ∑ n+1
n =0
et ∑ 2n+1 . Retrouver l’exercice précédent.
n =0

Solution :
+∞
0) Le théorème d’intégration terme à termes des séries s’applique si la série ∑ λ1
n =0 n
converge.

Dans le cas général, il ne s’applique pas. En attendant qu’on adjoigne aux prochains programmes un
nouveau théorème qui évitera de réfléchir, il faut se débrouiller tout seul, en se souvenant de cette
lettre de Tourgueniev à Tolstoï (1856) :
« Ceux qui s’attachent à des systèmes sont ceux qui, incapables d’embrasser la vérité toute entière,
tentent de l’attraper par la queue. Un système, c’est un peu la queue de la vérité, mais la vérité est
comme le lézard : elle vous laisse sa queue entre les doigts, et file, sachant parfaitement qu’il lui en
poussera une nouvelle en un rien de temps. »
1) Montrons d’abord l’existence des deux membres.
+∞ (−1)n
• La série ∑
n =0 λn
obéit au critère des séries alternées.

50
+∞
• La série de fonctions ∑(−1) .e λ
n =0
n − nx
obéit aussi au critère des séries alternées, pour chaque x > 0.

Sa somme f(x) est continue, car la série est uniformément convergente sur toute demi-droite [a, +∞[,
+∞
a > 0, car le reste Rn(x) = ∑(−1) .e λ
k = n +1
k − kx
tend uniformément vers 0 : | Rn(x) | ≤ e−λn+1x ≤ e−λn+1a .

−λ0 x
De plus, 0 ≤ f(x) ≤ e , donc f est intégrable.
2) Reste à montrer l’égalité des deux membres. Et là, il suffit d’utiliser les gendarmes !
2N +1 2N
Pour tout x et tout N : ∑ (−1)n.e−λnx ≤ f(x) ≤
n=0
∑(−1) .e λ
n =0
n − nx
.

Il reste à intégrer l’inégalité et à passer à la limite.


3) Applications :
+∞ (−1)n +∞ +∞ +∞ e−x
∑ ∑(−1)n.e−(n+1).x.dx =
1

n = 0 n +1
= ∫
0
n =0
∫0 1+e− x .dx = ∫ dt = ln 2.
0 1+t

+∞ (−1)n +∞ +∞ e−x
.dx = ∫ dt 2 = π .
+∞
∑ ∑(−1)n.e−(2n+1).x.dx =
1

n = 0 2n +1
= ∫ 0
n =0

0 1+e− 2 x 0 1+t 4
+∞ (−1)n +∞ +∞ +∞ e−x
∑ ∫ ∑(−1) .e
1
∫ .dx = ∫ dtα .
−(nα +1).x
= n .dx =
n =0 nα +1 0
n =0
0 1+e −α x 01+t

Exercice 13 : Pour tout n ≥ 1 et tout x ∈ R on pose un(x) = e−nx − 2.e−2nx .


+∞
1) Montrer que ∑u (x) converge pour tout réel x > 0, et calculer sa somme f(x) .
n =1
n

2) Montrer que chaque un est intégrable sur R+, ainsi que f .


+∞ +∞ +∞
3) Comparer ∫0
f(x).dx et ∑ ∫
n =1
0
un(x).dx . Expliquer ce résultat .

Solution : Les exercices précédants montrent que l’on peut parfois intégrer terme à terme une série
de fonctions sans que les théorèmes du cours ne s’appliquent. L’exercice qu’on va traiter fournit un
+∞ +∞ +∞ +∞
exemple très simple dans lequel ∫0
∑un(x).dx ≠
n =1
∑ ∫
n =1
0
un(x).dx .
+∞
1) Convergence simple. Pour chaque x > 0, ∑u (x)
n =1
n converge comme combinaison linéaire de
+∞ e−x e−2x
séries géométriques convergentes, et ∑u (x) = 1−e
n =1
n −x
−2 = 1 .
1−e−2x ex +1
On prendra garde toutefois que cette formule n’est pas vraie en 0 : un(0) = −1, tandis que f(0) = 1 .
2
2) Intégrabilités évidentes : un est combinaison linéaire de fonctions intégrables.
+∞ +∞ +∞
3) On a ∫
0
un(x).dx = 1 − 2 1 = 0. Donc
n 2n ∑ ∫
n =1
0
un(x).dx = 0.

+∞ +∞ +∞ +∞
Tandis que ∫ ∑u (x).dx = ∫
0
n =1
n
0
dx =
ex +1 ∫ 1
dt = ln 2 ( chgt de var. t = ex ). Et voilà !
t(t +1)
+∞ +1 +∞
Explication : ∫ 0
un(x).dx = 1
n ∫ 0
1−2s.ds = 1 , donc
2n ∑ ∫ 0
un(x).dx = +∞.

Il est curieux de constater que Maple s’y laisse prendre un instant…


> f:=x->exp(-x)/(1-exp(-x))-2*exp(-2*x)/(1-exp(-2*x));

51
( −x ) ( −2 x )
e 2e
f := x → ( −x )
− ( −2 x )
1−e 1−e
> int(f(x),x=0..infinity);ff:=simplify(f(x));int(ff,x=0..infinity);
0
( −x )
e
ff := ( −x )
e +1
ln( 2 )
> with(plots):u:=(k,x)->exp(-k*x)-2*exp(-2*k*x):
S:=(n,x)->sum(u(k,x),k=1..n):p:=n->plot(S(n,x),x=0..3,color=blue):
q:=plot(ff,x=0..3,color=red,thickness=2):display([q,seq(p(n),n=1..6)]);

9. Intégrales impropres à paramètres.

+∞
Exercice 1 : Étudier la fonction F(x) = ∫−∞

(t²+1)(t²+ x²)
.dt .

Solution : [ ENSAE 1991 ]


F(x) est l’intégrale d’une fraction rationnelle. Elle se calcule élémentairement ! Point n’est besoin de
se ruer sur les théorèmes du programme, dans lesquels s’embourba jadis un candidat maladroit !
Maple fournit les éléments de preuve :
> F:=x->int(t^2/((t^2+1)*(t^2+x^2)),t=-infinity..infinity);F(1);F(x);
1
π
2
− π x<0

 x−1

 2π x=0


 π
 0<x

 x+1
> convert(t^2/((t^2+1)*(t^2+x^2)),parfrac,t);
1 x2
− +
( x − 1 ) ( x + 1 ) ( t2 + 1 ) ( x − 1 ) ( x + 1 ) ( t2 + x2 )

Exercice 2 : Soit f une fonction R → R réglée et intégrable.


+∞
Montrer que sa transformée de Fourier F(x) = ∫−∞
f(t).e−ixt.dt est définie, continue et bornée sur R.

52
p
Montrer que, si f est de plus à décroissance rapide, i.e. si, pour tout entier p, f(t) = O(1/t ) en ±∞,

alors F est de classe C .

Solution :

Exercice 3 : Intégrale de Frullani.


e −e−bt + ∞ −at
Convergence et calcul de ∫0 t .dt pour a et b > 0.
+∞ e−at −e−bt
On pourra faire varier a, ou considérer F(x) = ∫ .e− xt dt .
0 t

Solution : Nous avons déjà rencontré cette intégrale au § 4.


e −e−bt
+ ∞ −at
1) Fixons b et considérons ∫0 t
.dt comme une fonction I(a) de a, alors
+∞
I’(a) = ∫
0
−e−at.dt = − 1 , donc I(a) = − ln a + cte. Comme I(b) = 0, I(a) = ln b – ln a.
a
Reste à montrer que le théorème de dérivation des IAP s’applique.
e−at −e−bt ∂f
Considérons la fonction f(a, t) = ; on a (a,t) = − e−at .
t ∂a
Les hypothèses H1, H2 et H3 s’appliquent à ces deux fonctions. Vérifions H3 :
Si K est un compact de R*+, K ⊂ [α, Α], alors, pour tout (a, t) ∈ K×R*+ ,
e−At −e−bt e−αt −e−bt e−αt −e−bt e−At −e−bt
≤ f(a, t) ≤ , donc | f(a, t) | ≤ max ( ,− )
t t t t
∂f
Et | (a,t) | = e−at ≤ e−αt . Voilà nos deux majorantes intégrables !
∂a
+∞ e−at −e−bt
2) La fonction F(x) = ∫ .e− xt dt est définie sur R+. Dérivons et réintégrons !
0 t
+∞
F’(x) = ∫ (e−(b+ x)t −e−(a+ x)t).dt = 1 − 1 , donc F(x) = ln(x + b) – ln(x + a). Il reste à faire x = 0.
0 b+ x a + x
Ici aussi, la dérivation sous l’intégrale reste à justifier.

+∞ e−t²
Exercice 4 : Montrer que F(x) = ∫−∞ x+ t
.dt est définie et continue sur ]0, +∞[.

Solution :

+∞ e−t
Exercice 5 : Etudier la fonction F(x) = ∫ 0 t+x
.dt . Domaine, propriétés, variations, graphe.
+∞
Montrer qu’au V(0+), F(x) = − ln x + ∫ e−t.lnt.dt + o(1).
0

Solution :

+∞ t − E(t) ∞
Exercice 6 : Montrer que la fonction F(x) = ∫0 t(t + x)
.dt est C sur R*+.
Limites et équivalents en +∞ et en 0+ ?

Solution : [ Oral 2005, RMS n° 479 ]

53
+∞
Exercice 7 : On considère la fonction : F(x) = ∫ 0
cos(xt).e−t².dt .

Domaine de définition ? Montrer que F est de classe C1 et vérifie une équation différentielle.
En déduire une expression de F(x) ( On admet que ∫R e−t².dt = π ).

Solution :

+∞
Exercice 8 : On considère la fonction : F(x) = ∫ 0
ch(xt).e−t².dt .

Domaine de définition ? Etablir que : F(x) = π ex²/4.


2

Solution :

+∞ x
Exercice 9 : Montrer l’identité de Legendre : ∫ 0
sin(xt).e−t² / 2.dt = e−x²/2 ∫ et² / 2.dt (∀x ∈ R)
0

+∞
Solution : La fonction F(x) = ∫ 0
sin(xt).e−t² / 2.dt est définie sur R.
En effet, pour tout x, la fonction f(x, .) : t → sin(xt). e−t² / 2 est continue et intégrable : | f(x, t) | ≤ e−t² / 2 .
∂f
De plus (x, t) = − t.sin(xt). e−t² / 2 est également intégrable. Les hypothèses (H1), (H) et (H3) sont
∂x
∂f +∞
, donc F est C et F’(x) = ∫ −tcos(xt).e−t²/ 2.dt .
1
vérifiées par f et
∂x 0
Une I.P.P. donne F’(x) = x.F(x) – 1, F(0) = 0.
Si l’on intègre cette équation différentielle par les méthodes classiques, on trouve le second membre.

Exercice 10 : Transformée de Fourier d’une gaussienne.


+∞
Soit a > 0. Convergence et calcul de F(x) = ∫
−∞
e−at²e−ixt.dt .

Solution : La fonction e−at² est intégrable et à décroissance rapide.


+∞ ∞
En vertu de ce qui précède, F(x) = ∫−∞
e−ixte−at².dt est définie, C sur R, et tend vers 0 en ±∞.
Reste à calculer F(x).
1ère méthode : équation différentielle.
+∞
F’(x) = ∫−∞
−ite−ixte−at².dt = ( IPP ) = − x F(x) .
2a
C’est une équation différentielle linéaire homogène d’ordre 1.
e−at².dt = π , F(x) = F(0). e 4a = π e 4a .
+∞ − − x² x²
Comme F(0) = ∫−∞ a a
2ème méthode : développement en série. Formellement :
+∞ +∞ +∞ (−ixt)n −at² +∞ (−ix)n +∞
∫−∞
e−ixte−at².dt = ∫−∞ ∑
n =0 n!
e .dt = ∑
n =0 n! ∫
−∞
t ne−at².dt
+∞ (−ix)2p +∞ +∞
= ∑ ∫ t 2pe−at².dt ( si n est impair, ∫ t ne−at².dt = 0 )
p =0 (2p)!
−∞ −∞

54
+∞ (−1) p x2p +∞ 2p −at² π (−1) p x2p
+∞
π e− 4a .

= ∑
p =0 (2p)!
2 ∫ t e .dt = … =
0 a ∑
p =0 p!(4a)
p =
a
+∞ +∞
car les intégrales ∫ 0
t 2pe−at².dt se ramènent à ∫
0
e−at².dt par des IPP.
Il reste à justifier l’intégration terme à terme des séries au moyen du théorème ad hoc.
3ème méthode : intégration complexe.
+∞ +∞ −a(t + ix )²− x² − x² +∞ −a(t + ix )²
F(x) = ∫
−∞
e−ixte−at².dt = ∫
−∞
ea 4a .dt = e 4a
−∞∫ e a .dt par mise sous forme canonique. Le
− x² +∞
π e− 4a .

changement de variable u = t + ix donne alors :
a
F(x) = 4a e ∫−∞
e−au².du =
a
Cette méthode est hélas erronée, car la variable d’intégration n’est pas réelle !
On peut cependant la rendre rigoureuse, mais il faut passer par l’intégration complexe, en
introduisant une intégrale curviligne convenable…

+∞ e−xt²
Exercice 11 : On considère la fonction F(x) = ∫ 0 1+t²
.dt .
1) Domaine de définition de F ?
2) Montrer que F est continue sur R+, de classe C1 sur R*+, et vérifie une équation différentielle.
3) En déduire la valeur de l’intégrale de Gauss I = ∫R e−t² dt .

e−xt²
Solution : 1) La fonction fx : t → est continue positive sur R+.
1+t²
• Si x ≥ 0, elle est intégrable, car 0 ≤ fx(t) ≤ 1 intégrable.
1+t²
• Si x < 0, elle tend vers +∞ en +∞, donc n’est pas intégrable.
Conclusion : F est définie sur R+.
e−xt²
2) La fonction f : (x, t) → obéit aux hypothèses
1+t²
(H 1) pour tout x ≥ 0, f(x, . ) est intégrable ;
(H 2) pour tout t ≥ 0, f(. , t) est continue ;
(H 3) majorante intégrable : ∀(x, t) 0 ≤ f(x, t) ≤ 1 .
1+t²
Par conséquent, F est continue sur R+ .
∂f −t².e−xt²
La fonction : (x, t) → obéit aux hypothèses :
∂x 1+t²
∂f −xt
(H 1) pour tout x > 0, (x, .) est intégrable (car ≤ e au V(+∞)) ;
∂x
∂f
(H 2) pour tout t ≥ 0, (. , t) est continue ;
∂x
e−at²
(H 3) majorante intégrable ∀a > 0 ∀(x, t) ∈ [a, +∞[×R+ 0 ≤ f(x, t) ≤ .
1+t²
+∞ t².e−xt²

1
Par conséquent, F est C sur [a, +∞[, donc sur R*+, et F’(x) = − .dt .
0 1+t²
+∞
Du coup, F(x) – F’(x) = ∫
0
e− xt².dt = I ( chgt de var t x = u ).
x
Ajoutons que F(x) → 0 en +∞, soit par convergence dominée (majorante intégrable),

55
+∞
soit par les gendarmes : 0 ≤ F(x) ≤ ∫0
e− xt².dt = I .
x
+∞ e−xs
Remarque : F est une transformée de Laplace : F(x) = ∫0 2(1+s) s
.ds .

3) Intégrons cette équation différentielle.


x
Equation homogène : F(x) = C.e .
x x −x
Variation des constantes : F(x) = C(x).e donne − C’(x).e = I , C’(x) = − I e , donc
x x
+∞ e− −t
x +∞ e
t
C(x) = I ∫ F(x) = I e ∫
x
.dt + A et .dt + A e
x t x t
− −
x +∞ e x +∞ e e +∞ −t
t t x

x ∫x
Je dis que e ∫ .dt → 0 en +∞, car 0 ≤ e ∫ .dt ≤ e .dt = 1 .
x t x t x
−t
x +∞ e
Comme F(x) → 0 en +∞, A = 0 et F(x) = I e ∫ .dt .
x t
+∞ e−t
Mais F est continue en 0 et F(0) = π . Donc π = I ∫
2
.dt = 2 I et I = π .
2 2 0 t 2

Exercice 12 : Intégrale de Poisson : P(x) = ∫(0,π) ln(x − 2x.cos θ + 1).dθ .


2

1) Domaine de définition de P ? Les questions 2 et 3 doivent être traitées indépendamment.


2) a) Montrer que P est continue sur R, dérivable en tout x ≠ ±1 ; calculer P'(x).
b) Trouver limx→+∞ P(x) − 2π.ln x . En déduire P(x).
2
3) a) Montrer que P est paire et vérifie : P(x ) = 2.P(x) et P(1/x) = P(x) − 2π.ln |x| .
b) Retrouver P(x).

Solution : [ Oral Centrale MP 2011, RMS n° 883 ]

π /2
Exercice 13 : On considère la fonction : F(x) = ∫ 0
ln(1+ x.sin ²θ).dθ .
1) Domaine de définition de F ? Continuité, dérivabilité ?
2) Montrer que F(x) = π.{ ln(1 + 1+ x ) − ln 2 } . Calculer F(−1) .

Solution :

π /2
Exercice 14 : Fonction de Wallis : W(x) = ∫0
sin x t.dt .
1) Domaine de définition de W ? Montrer que W est de classe C∞ sur son domaine.
2) Monotonie, convexité, limites ; calcul de W(0), W(1), W'(0) et W'(1) ; graphe ?
3) Relation entre W(x + 2) et W(x) ? Que dire de (x + 1).W(x).W(x + 1) ?
4) Équivalents de W(x) en −1+ et en +∞ ?

Solution :
Remarque : La fonction W est liée à la fonction Bêta : W(x) = 1 B( x+1 , 1 ).
2 2 2
On peut obtenir un d.a. à tous ordres de W en +∞, par la méthode de Laplace.

.dt = π e−x .
+∞ cos(xt) +∞ t.sin(xt)
Exercice 15 : Montrer les formules : (∀x > 0) ∫
0 1+t²
.dt = ∫
0 1+t² 2

56
Solution :

+∞
Arc tan(xt)
Exercice 16 : 1) Etudier F(x) = ∫
1+t²
0
.dt : définition, continuité, dérivabilité, limite en +∞.
+∞ Arc tan(xt)
2) Etudier la fonction G(x) = ∫ .dt . Calculer G’(x).
0 t.(1+t²)
+∞
En déduire G(x), puis la valeur de I = ∫ ( Arctant )².dt .
0 t

Solution :

+∞ ln(1+t x)
Exercice 17 : Domaine de déf. de F(x) = ∫0 1+t x
.dt . Limites en 1 et en −1 ; équivalents en ±∞.

Solution : [ Oral X 1989, RMS n° 68 ]


Cf mes feuilles d’exercices

+∞
Exercice 18 : Domaine de définition de F(x) = ∫0
dt . Continuité.
1+t +t x +1
Equivalent en 0, limite en + ∞.

Solution : [ Oral Centrale 1994, RMS n° 276, Oral X 1998, RMS n° 55 ]


Cf mes feuilles d’exercice.

+∞ ∞
Exercice 19 : Montrer que F(x) = ∫
0
cos(xt²).e−t.dt est C , mais que sa série de Taylor en 0
diverge.

Solution :

10. Fonctions eulériennes.

Exercice 1 : Stirling par convergence dominée.


+∞
1) Par le changement de variable y = x + t x , montrer que Γ(x + 1) = ∫0
y x.e− y.dy s’écrit


x
(x/e) . x f(x,t).dt , où f(x, t) = 0 si t ≤ − x et est à préciser sinon.
R
2) Déterminer la limite de f(x, t) lorsque x tend vers +∞, t étant fixé.
3) On suppose x ≥ 1. Montrer que
−t −t²/2
0 < f(x, t) ≤ (1 + t).e si t ≥ 0 , 0 < f(x, t) ≤ e si − x < t ≤ 0.
4) Conclure que Γ(x + 1) ∼ (x/e) . 2πx quand x → +∞.
x

Solution : Il existe de nombreuses preuves de la formule de Stirling. Les plus puissantes passent par
la méthode de Laplace. C’est une forme de cette méthode qui est ici proposée. Elle a l’inconvénient
d’être un peu dogmatique.
+∞
∫ ∫
x
Le changement de variable y = x + t x donne Γ(x + 1) = y x.e− y.dy = ( x ) x f(x,t).dt ,
0 e R
x
où f(x, t) = 0 si t ≤ − x , f(x, t) = ( 1 + t ) e−t x
si t ≥ − x .
x

57
Fixons t. Quand x tend vers +∞, f(x, t) = exp[ x ln (1 + t ) – t x ] tend vers exp − t² .
x 2
−t −t²/2
Supposons x ≥ 1. Je dis que 0 < f(x, t) ≤ ( 1 + t ).e si t ≥ 0 et 0 < f(x, t) ≤ e si − x < t ≤ 0.
( x −1).t²
A(t) = ln( 1 + t ) − x ln ( 1 + t ) + t ( x − 1) vérifie A’(t) = ≥ 0.
x (t +1)(t + x)
A est croissante sur R+ ; comme A(0) = 0, A(t) ≥ 0 pour t ≥ 0.
B(u) = ln(1 + u) ≤ u − u² pour −1 < u ≤ 0, par étude des variations ou développement en série.
2
On en déduit B( t ) ≤ 0 pour − x < t ≤ 0.
x
Le théorème de convergence dominée s’applique, avec une majorante intégrable
−t −t²/2
0 < f(x, t) ≤ ϕ(t) , où ϕ(t) = ( 1 + t ).e si t ≥ 0 , e si t ≤ 0.
Par conséquent, ∫
R
f(x,t).dt → ∫e R
.dt quand x → +∞.
−t² / 2

Exercice 2 : Formule de Gauss.


n
1) Montrer que, pour tout α > 0, Γ(α) = lim n→+∞ ∫ (1− nt) tα
0
n −1
.dt .
nα .n!
2) En déduire que Γ(α) = lim n→+∞ .
α(α +1)...(α + n)
3) Application : calculer Γ( 1 ).
2

Solution :
n α−1
1) Soit fn(t) = ( 1 − t ) t pour 0 < t ≤ n , fn(t) = 0 pour t ≥ n.
n
α−1 α−1
La suite fn(t) tend simplement vers exp(−t) t et vérifie (∀t > 0) 0 ≤ fn(t) ≤ exp(−t) t .
Le théorème de convergence dominée conclut.
n α 1
2) Calculons les intégrales In = ∫ (1− nt) tα
0
n −1
.dt = n ∫ (1−u) uα
0
n −1 .du .
1
Une IPP permet de calculer B(α, n+1) = ∫ (1−u) uα
0
n −1
.du . On trouve n!
α(α +1)...(α +n)
.

n
Attention, si l’on travaille directement ∫ (1− nt) tα
0
n −1 .dt , ça marche mal.

nα .n!
On en déduit que Γ(α) = lim n→+∞ .
α(α +1)...(α + n)
2n+1.n1/ 2.n! 22n+1.n1/ 2.(n!)²
3) Du coup, Γ( 1 ) = lim n→+∞ = lim n→+∞ = π via Stirling.
2 1.3.5...(2n+1) (2n+1).(2n)!

+∞ −x n
Exercice 3 : Soit a ≥ 0, In = ∫
a
e .dx (n ≥ 1). Equivalent de la suite (In) et nature de la série
+∞
∑I
n =1
n . ( Commencer par a = 0, a = 1, 0 < a < 1 et a > 1 ).

Solution : L’intégrabilité de la fonction ne pose aucun problème.

58
+∞ −x n

n
Si a = 0, In = e .dx = 1.Γ(1 ) = Γ(1+ 1 ) ( chgt de variable t = x ).
0 n n n
p!
Du coup, par Taylor-Young, In = Γ(1) + Γ'(1).1 + … + Γ(p)(1). p + O( 1p+1 )
n n n
Si a > 0, In est une fonction Γ incomplète ; cf mes notes manuscrites…

Problème 4 : Représentations intégrales de Ψ et Ω.


1 1−t x 1 1−(1−u) x
1) a) Montrer (∀x > 0) Ψ(x) = − γ − 1 +
x ∫
0 1−t
.dt = − γ − 1 +
x ∫
0 u
.du .
b) En déduire la formule de Stern :
+∞
x(x−1)...(x−n+1)
(∀x > 0) Ψ(x) = − γ − 1 + x +
x ∑
n =2
(−1)n−1.
n .n !
+∞ exp(−t)−exp(−xt)
2) Montrer que : (∀x > 0) Ψ(x) = − γ + ∫ 0 1−exp(−t)
.dt .

3) Par dérivation sous ∫ , établir la formule de Plana , où Ω = lnΓ :


+∞ 1 exp(−t)−exp(−xt)
(∀x > 0) Ω(x) = ∫
0 t
.[(x − 1).exp(−t) −
1−exp(−t)
].dt .
4) Montrer la formule de Cauchy-Binet :
+∞
(∀x > 0) Ω(x) = (x − 1) .ln x − x + 1 ln(2π) +
2 2 ∫
0
1 .( 1 −1− 1 ).exp(−xt).dt .
t 1−e−t t 2
[On pourra dériver deux fois les deux membres].
En déduire le développement asymptotique à tous ordres de Ω(x) en +∞.

Solution :

Exercice 5 : Intégrales se ramenant à Γ.


+∞ −ct b Γ(a /b)
1) Montrer que ∫0
e ta−1.dt est définie ssi a > 0 et c > 0, et vaut
b a /b
.
b .c
+∞ −t b
∫ e .dt = Γ(1 + b1 ) . Cas où b = 2 ?
2) En déduire que, pour tout b > 0,
0
1 1
3) Nature et calcul éventuel de ∫ ln 1t .dt et de ∫ u .(−lnu) .du .
0
x −1
0
a b

+∞
4) Montrer que : (∀x > 0) Γ(x) = ∫ t .(t − x).e .lnt.dt . x −1 −t
0
+∞
5) Montrer que : (∀x > 0) Γ(x) = ∫ exp(xt −e ).dt . t
−∞

Solution :

Problème 6 : Formule des compléments.


+∞ p.t p −1 − xt
p désigne un réel ∈ ]0, 1[ , et F(x) = ∫ 0 1+t
.e .dt .
1) Montrer que F est définie et continue sur R+ , de classe C1 sur R*+ ,
Γ(1+ p)
et vérifie : F'(x) − F(x) = − (∀x > 0) .
xp

59
+∞
2) En déduire : F(x) = Γ(1 + p).ex. ∫x
e−t.t p.dt .
+∞ t p −1
3) En conclure : Γ(p).Γ(1 − p) = ∫0 1+t
.dt .
p −1
1 t
4) On pose g(p) = ∫ 1+t .dt
0
.

t p −1
+∞ p
a) Montrer que ∫0 1+t .dt = g(p) + p1−1 .g( p−1 ) .
+∞ (−1)n +∞ (−1)n
p
b) Montrer que g(p) = ∑ et 1 .g( )= ∑ .
n =0 1+ np p −1 p −1 n = 0 (n+1)p −1
+∞ (−1)n −1
c) En déduire Γ(p).Γ(1 − p) = 1 + 2. ∑ .
n =1 n²p² −1

Solution :

Problème 7 : Exemples de transformées de Fourier.


1) Soit a un réel > 0. Montrer que, pour tout x ∈ R, la fonction t → ta−1.e−t.eixt est intégrable sur
+∞
]0, +∞[. On note Fa(x) = ∫0
ta−1.e−t.eixt .dt .

2) Montrer que Fa est de classe C1, et vérifie l’équation différentielle −( i + x ).F'a(x) = a.Fa(x).
En déduire Fa(x) = Γ(a).( x2 + 1 )a/2.exp(i.a.Arctan x).
3) En déduire les formules suivantes, pour tout x réel :
+∞ cos(xt) π . 1+ x² +1 et +∞ sin(xt) π . 1+ x² −1 .
∫0
e−t
t
.dt =
2 2 1+ x² ∫0
e−t
t
.dt =
2 2 1+ x²

Solution :
11. Transformation de Laplace, méthode de Laplace.

+∞
Exercice 1 : Soient f ∈ C(R+, R), a réel. On suppose que l’intégrale ∫ 0
f(t)e−at.dt .
+∞
Montrer que, pour tout x > a, l’intégrale ∫
0
f(t)e− xt.dt converge.

Solution : [ Oral Navale 2012, RMS n° 951 ]


+∞
Si l’intégrale ∫0
f(t)e−at.dt est absolument convergente, la règle de la majorante conclut.
Si elle est semi-convergente, une IPP est nécessaire…
Il en résulte que le domaine de définition d’une transformée de Laplace est une demi-droite.

+∞
Exercice 2 : Dans cet exercice, x est une variable réelle. On se propose de calculer I = ∫0
sint .dt .
t
+∞
1) a) Domaine de définition de la fonction F(x) = ∫
0
e− xt .sint .dt .
t
1
b) Montrer que F est de classe C sur R*+ ; calculer F'(x).
c) Limite de F en +∞ ? Conséquence ?
t
2) On note Si(t) = ∫ sinu u .du
0
pour tout réel t.

60
+∞
a) Montrer que G(x) = ∫ 0
e− xt.Si(t).dt est définie sur R*+ .
b) Montrer que x.G(x) → I quand x → 0+ . En déduire que F est continue en 0.
+∞
c) Applications : calculer I , ∫ 0
e−t .sint .dt , etc.
t

Solution :
1) Une transformée de Laplace.
+∞
a) Domaine de définition de F(x) = ∫ 0
e− xt .sint .dt .
t
La fonction t → sint est continue, et même somme d’une série entière sur R ; elle est bornée car
t
| sint t | ≤ 1. Pour tout x > 0 la fonction t → sint t e−xt est donc intégrable. Pour x = 0, elle est semi-
(n+1)π
intégrable. Pour x < 0, elle n’est ni intégrable, ni semi-intégrable, car ∫π
n
e− xt .sint .dt ne tend pas
t
vers 0 quand n → +∞. (poser t = nπ + θ …). Le critère de Cauchy est donc violé.
+∞
Conclusion : La fonction F(x) = ∫
0
e− xt .sint .dt est définie sur R+.
t
1
b) Montrons que F est de classe C sur R*+ , et calculons F'(x).
−xt ∂f −xt
La fonction f : (x, t) → sint e a une dérivée partielle en x, (x,t) = − sin t. e .
t ∂x
∂f
Pour tout x > 0 , f(x, .) et (x,.) sont continues par morceaux et intégrables.
∂x
∂f
Pour tout t > 0 , f( ., t) et (.,t) sont continues.
∂x
−at ∂f −at
Pour tout a > 0 , pour tout x ≥ a et tout t > 0, |f(x, t)| ≤ sint e et | (x,t) | ≤ sin t. e , qui sont
t ∂x
des majorantes intégrables.
1
Le théorème de dérivation des intégrales à paramètres s’applique et montre que F est C et
+∞ +∞ e(i− x)t +∞
F’(x) = − ∫
0
sint.e− xt.dt = − Im ∫
0
e(i− x).t.dt = − Im
i− x 0
1 =− 1 .
| = − Im x−i x²+1
c) Limite de F en +∞ et conséquences.
+∞
Elémentairement, |F(x)| ≤ ∫0
e− xt.dt = 1 , donc F(x) → 0 quand x → +∞.
x
On peut aussi utiliser le théorème de convergence dominée.

e− xt .sint .dt = π − Arctan x = Arctan 1 .


+∞
Conclusion : Pour tout x > 0 , ∫0 t 2 x

On aimerait faire tendre x vers 0 dans cette identité. Si sint était intégrable sur R+, il suffirait de
t
noter que F(x) est continue sur R+, par convergence dominée. Malheureusement, elle n’est que semi-
intégrable, et le programme n’autorise pas ce passage à la limite.
2) Une deuxième transformée de Laplace.
+∞
a) Montrons que G(x) = ∫0
e− xt.Si(t).dt est définie sur R*+ .
t
La fonction « sinus intégral » Si(t) = ∫ sinu u .du
0
est continue et même développable en série entière

sur R, et elle a une limite en +∞, donc elle est bornée sur R+.

61
−xt
Il en résulte aussitôt que, pour tout x > 0 , t → Si(t) e est intégrable.
b) Montrons que x.G(x) → I quand x → 0+ .
+∞ +∞
Un changement de variable donne xG(x) = ∫0
xe− xt.Si(t).dt = ∫
0
e−u.Si(u ).du .
x
−u −u
Or e Si( u ) tend simplement vers I.e quand x tend vers 0+.
x
−u −u
Et on a la condition de domination | e Si( u ) | ≤ || Si ||∞ e .
x
+∞
Donc x.G(x) tend vers ∫0
e−u.I.du = I.
Remarque : On peut aussi montrer cela directement par soustraction et cassage en deux.
c) Intégrons par parties !
+∞ −xt
F(x) = ∫
0
e− xt.Si'(t).dt = [e Si(t)] 0+∞ + xG(x) = xG(x) → I = F(0) quand x tend vers 0. cqfd

e− xt .sint .dt = π − Arctan x = Arctan 1 .


+∞
Conclusion : Pour tout x ≥ 0 , ∫0 t 2 x

∫0 .sint t .dt = π2 , ∫0 e−t .sint t .dt = π4 , etc.


+∞ +∞
En particulier

> with(inttrans);
[ addtable , fourier, fouriercos, fouriersin, hankel , hilbert, invfourier, invhilbert,
invlaplace , invmellin , laplace , mellin, savetable ]
> f:=t->sin(t)/t;laplace(f(t),t,x);
arctan 
sin( t ) 1
f := t →
t x
> with(plots):S:=x->int(sin(t)/t,t=0..x);S(x);
p:=plot(S(x),x=0..40,thickness=2):q:=plot(Pi/2,0..40,color=black):
display({p,q});
Si( x )

Exercice 3 : Soit f une fonction [0, +∞[ → C, réglée sur tout segment, vérifiant :
rs
(L) (∃r ∈ R) f(s) = O(e ) au V(+∞) .
+∞
Montrer que sa transformée de Laplace F(x) = ∫ 0
e− xs.f(s).ds est définie pour x > r, et vérifie :
f(0+)
F(x) = + o( 1 ) quand x → +∞ .
x x

Solution : Supposons pour commencer r = 0, autrement dit f bornée.


Alors f(s).exp(−sx) est intégrable pour tout x > 0.
+∞

x ∫
Le changement de variable xs = u donne : F(x) = 1 e−u.f(u ).du .
0 x

62
−u −u −u −u −u
e .f( u ) → f(0+).e quand u > 0, e .f( u ) → f(0) quand u = 0. De plus | e .f( u ) | ≤ || f ||∞.e .
x x x
+∞ +∞
Le théorème de convergence dominée conclut que ∫0
e−u.f(u ).du →
x ∫
0
e−u.f(0+).du = f(0+).
D’où le résultat.
+∞
On peut aussi calculer xF(x) – f(0+) = ∫
0
xe− xs.(f(s)− f(0+)).ds , casser en deux et majorer…
+∞
Dans le cas général, écrivons F(x) = ∫
0
e−(x−r)s.e−rs f(s).ds et appliquons ce qui précède à
g(0+) f(0+)
g(s) = f(s).exp(−rs). Alors F(x) = + o( 1 ) = + o( 1 ). Cqfd.
x−r x−r x x

+∞
Exercice 4 : Limite et équivalent de In = ∫ 0
dt .
(1+t 3)n
+∞ +∞ +∞
Nature et calcul éventuel des séries ∑In ,
n =1
∑(−1)n−1In et
n =1
∑ In .
n =1
n

Solution : [ Oral Mines 2013, RMS n° 582 ]


Nous allons traiter cet important exercice par plusieurs méthodes.
1) Limite.
La fonction fn(t) = 1 est continue positive et intégrable pour n ≥ 1, car fn(t) ∼ 13n au V(+∞)
(1+t 3)n t
Convergence simple : fn(t) ↓ f(t) , où f(t) = 0 si t > 0 , f(0) = 1.
Domination : 0 ≤ fn(t) ≤ f1(t) pour tout n et tout t.
On conclut que In ↓ 0 en vertu du théorème de convergence dominée. A noter qu’il s’agit de la
convergence dominée du pauvre, car il y a convergence uniforme sur tout segment [a, b] ⊂ ]0, +∞[ .
Voici d’ailleurs une preuve directe, élémentaire, de In ↓ 0, par cassage en deux.
ε +∞ +∞ I1
In = ∫
0
+ ∫ε ≤ ε+
(1+ε )
1
3 n −1 ∫ε dt ≤ ε +
1+t 3 (1+ε 3)n−1
.

ε > 0 étant fixé,


I1 → 0 quand n → +∞, donc
I1 ≤ ε, et In ≤ 2ε pour n assez grand.
(1+ε 3)n−1 (1+ε 3)n−1
2) Equivalent, 1ère approche. Une intégration par parties permet un calcul récurrent des In.
+∞
t +∞ + 3n +∞ t .dt = 3n ( In – In+1 ) .
3
In = ∫
0
dt =
(1+t 3)n (1+t 3)n 0 ∫0 (1+t3)n+1
n −1
Finalement 3n In+1 = (3n – 1) In , donc In = I1 ∏(1−31k ) .
k =1

On peut calculer I1, à la main ou avec Maple : I1 = 2π 3 .


9
n −1 n −1
Passons au log : ln In = ln I1 + ∑
k =1
ln(1− 1 ) = ln I1 +
3k ∑(− 31k +O(k1²))
k =1
n −1 n −1
= ln I1 − 1
3 ∑ 1k + ∑O(k1²) = ln I1 − 13 ln(n − 1) + γ + o(1) + SPSAC = − 13 ln n
k =1 k =1
+ A + o(1) .

SPSAC signifie : somme partielle d’une série absolument convergente. Posant K = exp A, il vient :
n −1
Conclusion : In = 2π 3 ∏(1−31k ) ∼ K , où K est une constante > 0.
9 k =1
3
n

63
n x.n!
Remarque : la formule de Gauss Γ(x) = limn→+∞ fournit la valeur de K.
x(x+1)...(x+n)

On trouve K = 2π 3 1 .
9 Γ(2/3)
+∞
3) Nature de la série ∑I
n =1
n .

Cette série diverge, en vertu de la règle de l’équivalent.


Mais on peut aussi montrer cette divergence directement.
+1 +1 +1
1ère idée : In ≥ ∫
0
dt ≥
(1+t 3)n ∫ 0
(1−t 3)n.dt ≥ ∫0
(1−t)n.dt = 1
n+1
+∞ +∞ +∞ +∞ +∞ +∞
2ème idée : Formellement ∑In =
n =1
∑ ∫
n=1
0
dt =
(1+t 3)n ∫ ∑
0
n =1
dt =
(1+t 3)n ∫
0
dt = +∞.
t3
Un argument d’associativité de bornes supérieures valide ce raisonnement.
4) Equivalent par méthode de Laplace.
+∞ +∞ −nln(1+t 3)
L’idée fondamentale de cette méthode consiste à noter que In = ∫0
dt =
(1+t 3)n ∫
0
e .dt .
3
Or exp(− n ln(1 + t )) est maximum pour t = 0.
C’est au V(0+) que se concentre la masse quand n tend vers +∞.
+∞ −n.t 3 Γ(1/3)

3
On peut donc penser que In ∼ e .dt = après changement de variable n t = s.
0 3.n1/3
On peut justifier cet équivalent de différentes façons.
1ère méthode : découpe à la Chasles.
3 3
Tout d’abord ∀t ≥ 0, ln( 1 + t ) ≤ t . D’autre part, par définition d’un équivalent en 0 :
3 3
∀ε ∈ ]0, 1[ ∃α > 0 ∀t ∈ [0, α] ( 1 − ε) .t ≤ ln( 1 + t ).
+∞ −n.t 3 +∞ −nln(1+t 3) α −n(1−ε)t 3 +∞
Donc ∫
0
e .dt ≤ In = ∫ 0
e .dt ≤ ∫ 0
e .dt + ∫α dt .
(1+t 3)n
Γ(1/3) +∞ −n(1−ε)t 3 +∞
dt ≤ Γ(1/3) +∞

3.n1/3
≤ In ≤ ∫
0
e .dt + 1
(1+α 3)n−1 ∫α 1+t 3
1 + 1
3.n1/3 (1−ε)1/3 (1+α 3)n−1 ∫0
dt .
1+t 3
1 Γ(1/3)
Par comparaison exponentielle-puissance, est négligeable devant
(1+α 3)n−1 3.n1/3
Γ(1/3) Γ(1/3)
Donc, pour n assez grand
3.n1/3
≤ In ≤ [ 1 + ε] .
3.n1/3 (1−ε)1/3
Or 1 + ε est de la forme 1 + ε’. La preuve est complète.
(1−ε)1/3
2ème méthode : changement de variable et convergence dominée.
+∞ es

3 s 1/3
Le changement de variable s = ln( 1 + t ) donne t = (e – 1) et In = e−ns. s 2/3 ds .
0 3(e −1)
+∞ eu / n +∞ eu / n
Puis ns = u donne In = ∫
0
e−u.
3n(eu /n −1)2/3
du = 11/3
3n ∫ 0
e−u.
(n(eu / n −1))2/3
du .

−u eu /n −u −2/3 −u −2/3
Or e tend simplement vers e .u et est majorée par e .u , qui est intégrable,
(n(e −1))2/3
u/n

d’intégrale égale à Γ(1/3). En vertu du théorème de convergence dominée,


+∞ eu / n +∞

0
e−u.
(n(e −1))2/3
du →
u/n ∫
0
e−u.u−2/3.du = Γ( 1 ) .
3

64
Γ(1/3) Γ(4/3)
Conclusion : In ∼ = quand n → +∞.
3.n1/3 n1/3

5) Développement asymptotique à tous ordres de In.


+∞
Revenons à l’expression : In = ∫
0
e−(n−1)s. s 1 2/3 ds .
3(e −1)
Elle fait apparaître In comme la transformée de Laplace évaluée en n − 1 de f(s) = 1 .
3(es −1)2/3
Or f(s) admet un développement asymptotique en 0+ de la forme :
f(s) =
a0 + a s1/3 + a s4/3 + a s7/3 + … + a sp−2/3 + O(sp+1/3) .
1 2 3 p
s 2/ 3

En réalité, on peut écrire : f(s) =


a0 + a s1/3 + a s4/3 + a s7/3 + … + a sp−2/3 + g(s) ,
1 2 3 p
s 2/ 3
p+1/3
où g(s) est O(s ) au V(0+) et aussi au V(+∞), car g(s) → 0 en +∞.
p+1/3
Par suite, il existe une constante B > 0 telle que, pour tout s > 0, | g(s) | ≤ B.s .
+∞ +∞ Γ(p+4/3)
Et ∫0
e−(n−1)s.g(s)ds ≤ B ∫ e−(n−1)s.s p+1/3.ds = B.
0 (n−1) p+4/3
= O( p1+ 4/3 ).
n
Par linéarité de la transformée de Laplace :
Γ(1/3) Γ(4/3) Γ(7/3) 7/3 Γ(p+1/3)
In = a0 + a1 + a2 + a3 s + … + ap + O( p1+ 4/3 ).
(n−1)1/3 (n−1)4/3 (n−1)7/3 (n−1) p+1/3 n
+∞ +∞ +∞
Exercice : Convergence et calcul de ∑(−1)n−1In ,
n =1
∑In.xn et
n =1
∑ In .
n =1
n

6) Avec Maple :
> with(plots):f:=(n,t)->1/(1+t^3)^n;
p:=n->plot(f(n,t),t=0..5,thickness=2):display([seq(p(n),n=1..6)]);

> int(f(1,t),t=0..infinity);
2
π 3
9
> J:=n->2*Pi*sqrt(3)/9*product(1-1/(3*k),k=1..n-1);
simplify(J(n));simplify(asympt(J(n),n),exp);
π 3 Γ n − 
1
n−1
 1 1  2  3
J := n → π 3  ∏  1 −
2

9 k=1  3 k   9
Γ( n ) Γ 
2
 
3
( 1/3) ( 4/3) ( 7/3) ( 103
/ ) ( 13/3 )

π 3   π 3   π 3   π 3  


1 4 1 14 1 140 1
π 3  
364 1
( 16/3 )
   1 
n n n n + O  
2 81 729 19683 177147 n 
+ + + + 
 2  n  
Γ  Γ  Γ  Γ 
9 2 2 2 2 Γ 
3
 3  3  3  3

65
> with(inttrans);f:=s->1/3*exp(s)/(exp(s)-1)^(2/3):
DL:=series(f(s),s=0,9);laplace(DL,s,n);
[ addtable , fourier, fouriercos , fouriersin , hankel , hilbert, invfourier , invhilbert,
invlaplace , invmellin , laplace , mellin , savetable ]
1 1 2 ( 1/3 ) 7 ( 4/3 ) 5 ( 7/3 ) 13 ( 10/3 ) 1 ( 13/3 )
DL := + s + s + s + s + s
3 s ( 2/3 ) 9 108 486 14580 21870
247 ( 16/3 ) 11 ( 19/3 ) ( 22/3 )
+ s + s + O( s )
44089920 13226976
4 14 140 364 728
π 3 π 3 π 3 π 3 π 3
2 π 3 81 729 19683 177147 1594323
+ + + + +
9 ( 1/3 )  2  ( 4/3 )  2  ( 7/3 )  2  ( 10/3 )  2  ( 13/3 )  2  ( 16/3 )  2 
n Γ  n Γ  n Γ  n Γ  n Γ  n Γ 
3 3 3 3 3 3
12844 108680
π 3 π 3
43046721 387420489 ( 22/3 )
+ + + laplace( O( s ), s, n )
( 19/3 )  2  ( 22/3 )  2 
n Γ  n Γ 
3 3
___________

Quelques identités tombées du ciel …


e−xs
+∞ +∞ sin(t − x) +∞
(∀x ≥ 0) ∫0 1+s² . ds = ∫x t . dt = ∫0 sin u.du
u+ x

∫0 sint t .e− xt.dt = π2 − Arctan x.


+∞
(∀x ≥ 0)

∫0 1−cos
+∞
(∀x > 0) t .e− xt.dt = − ln x + 1 ln( x2 + 1 ).
t 2
t .e− xt.dt = π − Arctan x + x.ln x − x .ln(1+ x²) .
∫0 1−cos
+∞
(∀x ≥ 0)
t² 2 2

∫0 1+t² .ds = ∫0 cos(x.tant).dt = π2 .e .


+∞ cos(xt) π /2 −x
(∀x ∈ R)

 π ( 1 − e ) pour x > 0
−x
2
+∞ sin(x.t)
(∀x ∈ R) ∫0 t(1+t²) .dt =  0 pour x = 0

 π (e −1)
x
pour x < 0
2
.dt = π .( 1 − e
+∞ sin(xt) −x/ 2
(∀x > 0) ∫ 0 t(1+t )
4 2
.cos x )
2

e−t².ch(tx).dt = π .e e−t².cos(tx).dt = π .e
+∞ +∞ −x²/4
∫ ∫
x²/4
(∀x ∈ R) et
0 2 0 2
+∞ −x²/2 x
(∀x ∈ R) ∫ 0
e−t² / 2.sin(tx).dt = e ∫e
0
t² / 2
.dt (Legendre)

.dt = π .Argshx
π /2
Arc tan(x.sint)
(∀x ∈ R) ∫0 sint 2
+∞ Arc tan(x / t) x
(∀x ≥ 0) ∫0 1+t² .dt = ∫0 tln²−t1.dt

66
ln(1+ xt)
x

2
(∀x > 0) .dt = 1 .ln( 1 + x ).Arctan x .
0 1+t² 2
+∞ ln(1+ xt²) x
(∀x ≥ 0) ∫0 t.(1+t²) .dt = − 12 ∫0 1ln−tt .dt .
∫ t .tln−1t .dt = ln xx++12 .
1
(∀x > −1) x
0

e −e−bt − xt
+ ∞ −at
(∀a, b > 0) (∀x > max(−a, −b)) ∫ .e dt = ln x+b .
0 t x+a
ln(1+ x.sin ²t).dt = π.ln 1+ 1+ x .
π /2
(∀x ≥ −1) ∫0 2
π
(∀a, b > 1) ∫0 ln(ba−−cos
cos x).dx = π.(Argcha− Argchb)
x
+∞ Arc tan(x +t)
(∀x ∈ R) ∫−∞ 1+t² .dt = π.Arctan 2x .
∫−∞ 1+t 4 .dt = π.Arctan(x 2 + 1) − π4² pour x ≥ 0
+∞ t.Arc tan(x.t)
(∀x ∈ R)

π.Arctan(x 2 − 1) + π² pour x ≤ 0.
4
+∞ +∞
(∀x ≥ 0) ∫0
exp−(t + x² ). dt = π .e−2x &
t t ∫ 0
exp−(t² + x ).dt = π .e−2

x

−xt a
+∞ e +∞
(∀x, a > 0) ∫ .dt = Γ(1 + 1 ) ∫ t −1/ a.e x −t.dt
0 1+t a a x
+∞
(∀x > 0) (∀u ∈ R) ∫
−∞
e−πxt².e−2iπut.dt = 1 e−πu² / x .
x
+ ∞ −t e π
(∀x ∈ R) ∫0 t
.eitx.dt = 4
1+ x²
[cos Arc tan x + i.sin Arc tan x ].
2 2
+∞ +∞
(∀x > 0) ln x =
x ∫ 0
lnt .dt
(t + x)²
et ln²x =
2 ∫ 0
( 1 − 1 ).lnt.dt
t +1 t + x)
_____________

67

Vous aimerez peut-être aussi